You are on page 1of 77

A-PDF Merger DEMO : Purchase from www.A-PDF.

com to remove the watermark

1
Solutions to sample quiz problems and assigned problems
Sample Quiz Problems
Quiz Problem 1. Prove the expression for the Carnot efficiency for a perfectly reversible Carnot cycle using an
ideal gas.
Solution: The ideal Carnot cycle consists of four segments as follows (1) An isothermal expansion during which
heat QH is added to the system at temperature TH ; (2) an adiabatic expansion during which the gas cools from
temperature TH to TC ; (3) An isothermal compression during which heat QC is extracted from the system to a cold
reservoir at temperature TC ; (4) Adiabatic compression during which the temperature of the gas rises from TC to TH .
The efficiency of the engine is given by = W/QH , where W is the useful work done by the system and QH is the
heat added to the system. This is a thermodynamic cycle provided it is carried out reversibly (i.e. kept very close
to equilibrium), and then the internal energy is the same at the beginning and the end of the cycle. In that case,
dU = 0, and Wtotal = QH QL , so we can also write the efficiency as
=1

QL
TL S
TL
=1
=1
QH
TH S
TH

(1)

The introduction of Entropy by Clausius greatly simplified understanding of engines.

Quiz Problem 2. Show


ln(
)) reduces to
P that in the eigenfunction basis, the von Neuman entropy S = kB tr(
the Gibbs form S = kB i pi ln(pi ) where the sum is over all eigenstates.
Solution. See Eq. (20) of Lecture notes.

Quiz Problem 3. Use Stirlings approximation to show that,


 
N
ln(
) N [pln(p) + (1 p)ln(1 p)],
n

where

p = n/N

Solution. Using Stirlings approximation we have,


 
N
ln(
) = ln(N !) ln(n!) ln(N n)! = N ln(N ) N nln(n) + n (N n)ln(N n) + (N n)
n

(2)

(3)

which reduces to
 
N
ln(
) = N ln(N ) nln(n) (N n)ln(N n).
n
Defining p = n/N gives,
 
N
ln(
) = N ln(N ) pN ln(pN ) N (1 p)ln(N (1 p)) = N [pln(p) + (1 p)ln(1 p)]
n

(4)

(5)

Quiz Problem 4. If S = kB N [pln(p) + (1 p)ln(1 p)], by doing a variation with respect to p find the value
of p that gives the maximum entropy. Demonstrate that it is a maximum by showing that the second derivative with
respect to p is negative.
Solution. Taking a derivative with respect to p gives,
S
= kB N [ln(p) ln(1 p)]
p

(6)

2
The extrema occur at values of p = p where this expression is zero, which yields p = 1/2. To determine whether
this is a minimum of maximum, we take the second derivative to find that,
1
2S
1
= kB N [ +
]p=p = kB N
2
p
p 1p

(7)

Since the second derivative is negative the extremum at p = 1/2 is a maximum. We have thus found the value of
the probability at which the entropy is a maximum.

Quiz Problem 5. Use the Master equation to prove the second law of thermodynamics, i.e. in a closed system
dS/dt 0.
Solution. see lecture notes page 10.

Quiz Problem 6. Give three examples of systems where the ergodic hypothesis fails. Explain why it fails in these
cases.
Solution. Non-interacting classical gas, as in the absence of interactions the systems is non-chaotic. Coupled
harmonic oscillators with no other interactions, as the normal modes do not share energy. Any system with spontaneous symmetry breaking at low temperature, e.g. magnetic systems. Once in the up magnetized state at low
enough temperature the system stays in that state. Similarly once a crystal has formed from the gas phase at low
temperature it almost never transforms to another crystal phase.

Quiz Problem 7. Why is the ideal gas law still a good starting point for high temperature gases, even though a
non-interacting gas is non-ergodic?
Solution. Even very small interactions can make the system chaotic. These interactions have little effect on
the equilibrium thermodynamic behavior though they are critical to the transport properties.

Quiz Problem 8. Using pi = eEi /Z in the Gibbs


P form for the entropy, show that F = kB T ln(Z), where
F = U T S is the Helmholtz free energy. Here Z = i eEi is the canonical partition function.
Solution. The Gibbs formula for the entropy is
S = kB

pi ln(pi ).

(8)

Using the Boltzmann probability in the canonical ensemble pi = exp(Ei )/Z, we have,
S = kB

pi [

where we used

pi = 1, U =

U
Ei
ln(Z)] =
+ kB ln(Z)
kB T
T

so

U T S = kB T ln(Z),

(9)

pi Ei

Quiz Problem 9. If x = e(Ei Ej ) , so that wji f (x). Show that the Metropolis algorithm is given by
f (x) = min(x, 1).
Solution With this notation, the Metropolis algorithm is: If x > 1, wji = 1, and if x < 1, wji = x. This can be
put in one equation as wji = f (x) = min(1, x).

Quiz Problem 10. If x = e(Ei Ej ) , so that wji f (x). Show that detailed balance is satisfied for any f (x)
satisfying f (x)/f (1/x) = x. Moreover, show that f (x) = x/(1 + x), the heat bath algorithm, is one such solution.
Solution Since wij = 1/wji , if f (x) = wji , and if f (1/x) = wij , then detailed balance is satisfied provided
f (x)/f (1/x) = x. For the heat bath case,
f (x)/f (1/x) = (x/1 + x)/(1/x/(1 + 1/x)) = x

(10)

Quiz Problem 11. For a monatomic interacting classical gas, with interactions that only depend on the particle
co-ordinates, derive the Maxwell Boltzmann distribution of velocities and show that the average kinetic energy is
given by < KE >= 3N kB T /2.
Solution. See page 23 of the notes.

Quiz Problem 12. Using the fact that E 2 =< E 2 > < E >2 = kB T 2 CV show that E/E is proportional 1/N 1/2 .
Solution. See page 28 of the notes.

Quiz Problem 13. Write down the central difference form of the second derivative. Using this expression for the
acceleration, write down the basic form of the Verlet algorithm for Molecular dynamics.
Solution. See. page 21-22 of the notes.

Quiz Problem 14. Given that the virial is defined to be G =


derivative is expected to be zero at long times.

p~k ~rk . Explain why the average of its time

Solution. The average of the time derivative of the virial gives,


Z
dG
1 dG
<
>=
dt = (G( ) G(0))/
dt
0 dt
where the average is taken over the time interval . Since G is bounded, as goes to infinity, the average of the time
derivative of G goes to zero.

Quiz Problem 15. The ground state structure of Argon is a fcc crystal. If a MD simulation is carried out where
a high temperature structure is quenched to a low temperature where the fcc structure is the equilibrium state. If
the starting high temperature state in your MD simulation is a liquid phase, do you expect the simulation to find the
correct fcc state? Explain.
Solution. For most particle systems it is difficult to find the low temperature ground state, so the fcc single crystal
state is usually not found. In experiments a polycrystal is quit often found but it is harder to make single crystals.
In MD simulations, the timescale is very short so unless special conditions are applied, the low temperature state is
a frozen glass state.

Quiz Problem 16. In a MC calculation of the ferromagnetic Ising model on a square lattice, you calculate the
magnetization as a function of temperature as accurately as you can. However your simulations remain rounded
at the critical point, instead of showing the behavior |Tc T | expected in the thermodynamic limit. Explain the
physical origins of the rounding that you observe. Your experimental colleague does a measurement of a model Ising
magnet that consists of quantum dots with around 1000 spins in each quantum dot. She also observes a rounding
of the behavior of the magnetization. Provide some reasons why her magnetization experiment also gives a rounded
behavior near the critical point.
Solution. Three reasons for rounding of the transition are finite size effects, sample inhomogeneity and nonequilibrium effects. Non-equilibrium effects occur when the system is not fully equilibrated. In experiments all three
effects occur, while in simulations the sample inhomogeneity effect is avoided.

Quiz Problem 17. Derive the relation,


P V = N kB T +

1
3

internal
X

~rk F~k

(11)

Explain the physical origin of each of the three terms. This relation is not always true for periodic boundary conditions, though rather fortuitiously it is true for periodic boundaries provided the interaction potential is a pair potential.
Solution. See page 23-24 of the notes.

Quiz Problem 18. Show that


(E)2 = kB T 2 CV

(12)

Solution. See page 28 of the notes.


Solutions to assigned problems
Problem 1. Find the relation between pressure and volume for an ideal gas under going an adiabatic process.
Solution:. In an adiabatic process no heat is added to the system, so dU +dW = 0, where dW = P dV . Combining
the ideal gas law and the equipartion theorem, we have
dU = N kB dT = d(P V ) = (V dP + P dV )

(13)

where = DOF/2. Using the first law, we then have,


P dV = (V dP + P dV ) or

dP
dV
= ( + 1)
P
V

(14)

Integrating gives,
P
V0
= ( )1+1/
P0
V

(15)

which may be written in its most common form,


P V = constant where =

+1

(16)

For a mono-atomic ideal gas in three dimensions there are three translational degrees of freedom, so DOF = 3,
= 3/2, and = 5/3.

Problem 2: Derive the ideal monatomic gas law using kinetic theory. You may use the equipartition result
U = 23 N kB T .
Solution: The internal energy of an ideal gas is purely kinetic energy, so that,
U=

1X
1
3
N kB T =
m < [(vxi )2 + (vyi )2 + (vzi )2 ] >= N m < ~v 2 >
2
2 i
2

(17)

The pressure is calculated by considering a particle incident normally on a perfectly reflecting wall,
Fx = max = m

px
2mvx
=
t
t

(18)

The time taken for the particle to strike the wall again is t = 2L/vx , so that
P i
P
m i (vxi 2 )
2mvx2
i Fx
Fwall =
so that P =
=
2L
A
AL
P i 2
Noting that V = AL, with i (vx ) = 13 < ~v 2 > and combining (12) and (14) yields,
PV = m

(19)

X
(vxi )2 = N kB T

(20)

Problem 3. Find the degeneracy (E, N ) for N a spin 1/2 Ising system with Hamiltonian H = h

Si .

Solution: The energy of a spin state depends only on the number of up spins Nu , and the number of down spins
Nd , but not their arrangement. We then have,
h
E = (Nu Nd ),
2

with degeneracy

(E) =

N!
Nu !Nd !

(21)

We also have N = Nu + Nd , so that


h
E = (2Nu N )
2

or Nu =

N
E
E
N
, Nd =

2
h
h
2

(22)

Since entropy is the kB ln((E)), we use Stirlings approximation to write,


ln((E)) = N lnN N Nu lnNu + Nu Nd lnNd + Nd = N lnN Nu lnNu Nd lnNd

(23)

It is useful to introduce the fraction f = Nu /N , so that Nd /N = 1 f and


ln() = N lnN f N ln(f N ) (1 f )N ln[(1 f )N ] = N [f ln(f ) + (1 f )ln(1 f )]

(24)

which shows that the entropy in this model is extensive. In terms of f , E = (N h/2)(2f 1). We define  = E/N , so
that f = 1/2 /h. Finally the entropy is,
S = kB ln((E)) = kB N [(1/2 /h)ln(1/2 /h) + (/h 1/2)ln(/h 1/2)]

(25)

6
Problem 4. Consider a random walk on a one dimensional lattice where the probability of stepping to the right
or to the left is 1/2. Find the probability that after N steps the walker is at position x lattice units from the starting
position. Use Stirlings approximation to show that for large N the probability distribution is a Gaussian in x. Find
the average mean square distance the walker is from the origin as a function of N .
Solution: Let us define the number of steps that the student takes to be N = t/t. The number of steps taken in
the positive x-direction, n, and the number taken in the negative x-direction, m, must add up to N , so that,
N =n+m

(26)

The x position of the student at time t = N t is given by x = dx, where


d = (n m).

(27)

But what is the probability that the student reaches this position at time t?
P (d, N ) =

N!
1 N!
1
= N
N
2 m!n!
2 ((N + d)/2)!((N d)/2)!

(28)

Stirlings approximation:
Log(n!) nln(n) n

(29)

We then have,
Ln(P (d, N )) N Ln(2) + N Ln(N ) N

N +d
Ln((N + d)/2) + (N + d)/2
2

N d
)Log((N d)/2) + (N d)/2
2

(30)

This reduces to,


Ln(P (d, N )) N Ln(2) + N Ln(N )

N +d
[Ln(N ) + Ln(1 + d/N ) Ln(2)]
2

N d
)[Ln(N ) + Ln(1 d/N ) Ln(2)]
2

(31)

Defining y = d/N and expanding for small y yields,


ln(P (d, N )) =

N
N
1
1
[(1 + y)ln(1 + y) + (1 y)ln(1 y)] [(1 + y)(y y 2 ) + ...) + (1 y)(y y 2 ) + ...)] (32)
2
2
2
2

To leading order this yields


ln(P (d, N )) N y 2 =

d2
N

(33)

or,
2

P (d, N ) ed

/N

ex

t/(x)2 t

(34)

Comparing to the solutions to the continuum diffusion equation in one dimension, this shows that the diffusion
constant is related to the stochastic parameters x and t through,
D (x)2 /t.

(35)

From the probability distribution (28), it is easy to see that the average displacement of an ink particle is < x >= 0,
but the typical distance from the origin is (< x2 > < x >2 )1/2 N 1/2 x.

Problem 5. Prove the central limit theorem for a set of N independent subsystems that each have an energy Ei
drawn from a Gaussian distribution with standard deviation . You will need to use the integral,
Z
2
b2

eax +bx dx = ( )1/2 e 4a


(36)
a

Solution: Consider any intensive thermodynamic variable Xi (e.g. E/N ) that has a different value in each of N
subsystems. Consider each subsystem to have the same volume and that each subsystem is statistically equivalent.
Each subsystem is assumed to have a statistical behavior that is Gaussian distributed, so that,
P1 (Xi ) = 2 2

1/2

Xi2

e 22

(37)

where we assumed that each subsystem has the same statistical variations, quantified by . We would like to calculate
the statistical behavior of the quantity
N
1 X
Xi
N i=1

XN =

(38)

The statistical behavior of X N is given by,


PN (X N ) = 2 2

N/2

1
2

Xi2

dXi e 22 (

Xi N X N )

(39)

Using the delta function representation,


() =

eiy dy

(40)

we find,
PN (X N ) = 2 2

N/2

1
2

dy

Xi2

dXi e 22 eiy(

Xi N X N )

(41)

which is equivalent to
PN (X N ) = 2


2 N/2

dy iyN X N N
e
[I]
2

(42)

where
Z

I=

X2

dXe 22 eiyX = 2 2

1/2

2 y2
2

(43)

Finally,
Z

PN (X N ) =

2
2
dy iyN X N N 2 y2
1
2
e
e
=
eX N /2N
1/2
2
(2N )

(44)

where N = /N 1/2

Problem 6. Find the entropy of mixing


Pm for a system consisting of m different atom types, with ni of each atom
type and with the total of atoms, N = i ni .
Solution: The number of ways of arranging the system is given by the multinomial,
({ni }) =

N!
n1 !n2 !...nm !

(45)

8
Using Stirlings approximation we find,
ln() = N lnN N [

X
X
(ni ln(ni ) ni )] = N lnN N [ (N pi ln(N pi ) N pi )]
i

where pi = ni /N . Using

(46)

pi = 1 this simplifies to,


S = kB ln() = kB N

pi ln(pi )

(47)

which is the same form as the Gibbs entropy, though the interpretation of pi is different here as it is not the probability
of an individual many body state.

Problem 7. Derive Stirlings approximation ln(N !) = N ln(N ) N .


Solution. Expanding the logarithm of N ! as a sum we write,
ln(N !) =

N
X
i=1

Z
ln(i)

ln(x)dx = [xln(x) x]|N


1 = N ln(N ) N + 1

(48)

In the large N limit the constant one is negligible.

Problem 8. Find the entropy of a system of N non-interacting gas particles in a box of volume V = Ld , where d
is the dimension. Consider two cases: Where the particles are distinguishable; and where they are indistinguishable.
Is the entropy extensive in both cases? Assume that all configuration of the particles have the same energy, so we
can ignore the kinetic energy term.
Solution
In the case where all of the particles are distinguishable, the number of available configurations is V N as each
particle can be at any position in the volume. The entropy is then S = kB N ln(V ), which is not extensive. Gibbs
realized that identical particles are not distinguishable so the naive calculation of the number of states V N is not
valid. Instead, we must order the co-orindates to eliminate overcounting of indistinguishable particles.
Consider a set of particles with co-ordinates ~r1 ...~rN . If we order one component of these co-ordinates, say the
x-component so that x1 x2 x3 ... xN then the co-ordinates of the particles are always different hence ensuring
that there is no overcounting. The integration over the spatial co-ordinates then becomes,
Z L
Z x1
Z xN 1
VN
LN
d1
=
(49)
=L
dx1
dx2 ...
dxN = L(d1)N
N!
N!
0
0
0
The entropy of indistinguishable particles is then,
S = kB N [ln(V ) ln(N ) + 1] N kB ln(V /N )

(50)

which is extensive for V /N finite.

Problem 9. Prove the central limit theorem holds for any distribution provided it has a finite standard deviation.
i.e. extend the solution to problem 5 to arbitrary distributions that have a finite second moment.
Solution We consider that the statistics of an extensive quantity X obeys a distribution g(X) that has mean X
and standard deviation . Without loss of generality, we can set X = 0 by a simple shift of X. The statistical
behavior of X N is given by,
Z Y
X

2 N/2
PN (X N ) = 2
dXi g(xi )(
Xi N X N )
(51)

9
Using the delta function representation,

1
() =
2

eiy dy

(52)

we find,
PN (X N ) = 2 2

N/2

1
2

dy

dXi g(Xi )eiy(

Xi N X N )

(53)

which is equivalent to
PN (X N ) = 2


2 N/2

dy iyN X N N
[I]
e
2

(54)

where
Z

I=

dXg(X)eiyX

(55)

Now X is extensive, so the exponential is a rapidly oscillating function and the integral is dominated by the region
near y = 0. We then expand the exponential to find,
Z
Z
1
iyX
I=
dXg(X)e

dXg(X)(1 iyX y 2 X 2 + ...)


(56)
2

Assuming that g(X) is symmetric, this reduces to,


Z
1 2 2
1
I=
dXg(X)eiyX (1 y 2 2 + ...) e 2 y
2

Finally, this is of the same form as that of Problem 5 and hence the result there applies,
Z
2
2
dy iyN X N N 2 y2
1
2
PN (X N ) =
e
e
=
eX N /2N
1/2
2
(2N
)

(57)

(58)

where N = /N 1/2

Problem 10. Which of the following is an exact differential? a) dU (x, y) = 2xdx + 3xydy; b) dU (x, y) =
(1/y)dx + (x/y 2 )dy; c) dU (x, y) = ydx + xdy ? For these three examples, set dU = 0 and solve the differential
equations to find y(x). In cases where the differential is exact, find U (x, y).
Solution:
a) We have
U
= 2x;
x

and

U
= 3xy
y

(59)

so that
2U
2U
= 1/y 2 6=
= 3y
xy
yx

(60)

Therefore dU IS NOT an exact differential.


b) We have
U
1
=
;
x
y

and

U
x
= 2
y
y

(61)

10
so that
2U
1
1
2U
= 2 =
= 2
xy
y
yx
y

(62)

U
= y;
x

(63)

Therefore dU IS an exact differential.


c) We have
and

U
=x
y

so that
2U
2U
= 1 6=
=1
xy
yx

(64)

Therefore dU IS NOT an exact differential.


Now we solve the three cases and if possible find U .
a) We find,
dy
2
=
,
dx
3y

with solution

y=

4
4
(c x)1/2 ; or y = (c + x)1/2
3
3

(65)

b) In this case,
y
dy
= ,
dx
x

with solution

y = cx

(66)

This is an exact differential, where U (x, y) = x/y.


c) The solution for dU = 0 is the same as b), but for finite U , it is not an exact differential so we cannot find
U (x, y) to satisfy the equation.

Problem 11. By using the fact that S is extensive show that








S
S
S
N
+V
+U
=S
N V,U
V N,U
U N,V

(67)

and hence that N = U + P V T S.


Solution. Taking a derivative with respect to lambda of the homogeneous equation,
S(U, V, N ) = S(U, V, N )
and then setting = 1 yields Eq. (68). From the fundamental thermodynamic relation we have,






1
P

S
S
S
dS = dU + dV dN =
dU +
dV +
dN
T
T
T
U N,V
V N,U
N V,U

(68)

(69)

Using the relations inferred from this equation in Eq. (68) yields N = U + P V T S.

Problem 12. Using the fundamental thermodynamic relation and N = U + P V T S prove the Gibbs-Duhem
relation.
Solution. From the fundamental thermodynamic relation we have,
dU = T dS P dV + dN = d(T S P V + N )

(70)

From the equality of the last two expressions we deduce


SdT V dP + N d = 0

(71)

11

Problem 13. From the fundamental thermodynamic relation show that,







V
=
.
P S,N
N S,P
Solution: Since the independent variables are S, P, N , we choose the enthalpy and write,






H
H
H
dH = T dS + V dP + dN =
dS +
dP +
dN
S P,N
P S,N
N S,P

(72)

(73)

hence,


H
P


=V;

S,N

H
N


= .

(74)

S,P

By doing a derivative with respect to N of the first equation in (75) we find,



 !



H
V
=
.
N P S,N
N S,P

(75)

S,P

while a derivative with respect to P of the second equation in (75) yields




 !


=
.
P N S,P
P S,N

(76)

S,N

Because dH is an exact differential, the order of the derivatives of H in Eq. (76) and (77) does not matter so these
expressions are equivalent hence proving the Maxwell relation (73).

Problem 14. From the fundamental thermodynamic relation show that,




 2 
CP
V
= T
.
P T,N
T 2 P,N

(77)

Solution: Since the independent variables in the expression are T, P , we consider the Gibbs free energy G(T, P, N ),
so that,






G
G
G
dG = SdT + V dP + dN =
dT +
dP +
dN
(78)
T P,N
P T,N
N T,P
hence,


G
T


= S;
P,N

G
P

V
T

= V.

(79)

T,N

This leads to the Maxwell relation,




S
P


=
T,N

(80)
P,N

Now take a temperature derivative at constant pressure of this equation, to find,


!


 2 

S
V

=
T P T,N
T 2 P,N
P,N

(81)

12
We can change the order of the derivatives of the expression on the left hand side, and use the fact that CP =
T (S/T )P to write,


 2 

 2 

CP
V
V
(CP /T )
; or
(82)
=
= T

2
P
T
P
T 2 P,N
P,N
T,N
T,N
which solves the problem.

Problem 15. The equipartition theorem states that for a classical particle system of volume V with N particles,
described by Hamiltonian H, the following relations hold,
< si

H
>= kT ij
sj

(83)

where si can be either the spatial or momentum co-ordinate. Using the canonical partition function, carry out the
integrals explicitly to verify this theorm for a system described by the quadratic Hamiltonian,
H=

X p2
X1
i
+
kqi2 .
2m
2
i
i

(84)

Find the internal energy of this system as a function of temperature. Is this consistent with the equipartion theorem?
Solution We need to evaluate the following quantities explicitly,
< qi

H
>= k < qi qj >;
qj

< pi

1
H
< pi pj >
>=
pj
m

(85)

It is convenient to define,
(l)
Ii

Z
=

l kqi2 /2

(qi ) e

(l)
Ji

dqi ;

Z
=

(pi )l epi /(2m) dpi

(86)

It is then evident that,


R
< qi qj >=

dq q eH
R i j
deH

(87)

Reduces to,
(1) (1)

< qi qj >=
(

Ii Ij
(0)

,
(0)

(2)

f or i 6= j;

Ii Ij

< qi qj >=

Ii

(0)

f or i = j;

(88)

Ii

Now note that Ii l) and Ji l) are zero for l odd, and also that they are independent on the index i, we thus find,
R 2 kq2 /2
q e
dq
< qi qj >= ij R
(89)

2 /2
kq
e
dq

Now we use the standard integrals,


Z

q 2n eq dq =

(2n 1)!! 1/2


( )
(2)n

(90)

where (2n 1)!! = (2n 1)(2n 3)... and is one for n = 0 or n = 1. Using this result verifies the equipartition formula.
The calculation for the momentum is essentially the same.
The internal energy is < H > and since we have found that k < qi2 >=< p2i > /m = kB T , the internal energy
of this problem is 3N kB T . This is consistent with the equipartition formula that assigns kB T /2 for every quadratic
degree of freedom. Here there are six degrees of freedom of that type.

13

Problem 16. You discover a new state of matter that has equation of state and internal energy given by,
P = AT 3 /V,

U = BT n Ln(V /V0 ) + f (T )

(91)

where B, n, V0 are constants while f (T ) only depends on the temperature. Find a numerical value for n, and find a
relation between the constants B and A.
Solution It is convenient to define an entropy that is a Legendre transform of the conventional entropy so that,
SL = S

U
T

(92)

We then have,
dSL =

P
U
dV + 2 dT
T
T

(93)

where we used dS = dU/T + P dV /T . The Maxwell relation derived from dSL in combination with the functions given
in the problem definition yield,
AT 2

f (T )
(
)=
(BT n2 ln(V /V0 ) +
)
T V
V
T2

(94)

This equation is satisfied provided n = 3 and B = 2A.

Problem 17. From the fundamental thermodynamic relation show that (here we use = 1/T )




E
N
=
T
T V,
T,V

(95)

Solution: Since the independent variables are , V, , we start with the entropy form of the fundamental relation,
dS = dU + P dV dN

(96)

where = 1/T . Since the independent variables in the desired expression are , V, , we define, Y = S U +()N ,
so that,






Y
Y
Y
dY (, V, ) = U d + P dV + N d() =
d +
dV +
d()
(97)
V,
V ,
,V
hence,



= U ;
V,

Y
()


= N,

(98)

,V

that leads to the Maxwell relation,




U
()


=
,V


(99)
V,

Since is constant for the left hand side derivative, we can take it outside. For the right hand derivative, we use the
chain rule to find






1 U
N
T
=
(100)

,V
T ,V ,V
which reduces to the equation posed in the problem.

14

Problem 18. From the fundamental thermodynamic relation show that










T
N
(S/N )
P
1
2
=
and
=
T 2 E,V
E ,V

T ,N
T,N

(101)

where = N/V is the number density.


Solution: For the first equation, the variables held constant are and E, so we use the entropy form of the
fundamental relation, dS = dU + P dV dN . We also define Y = S + ()N to find a relation that has as
an independent variable. We then have,






Y
Y
Y
dY (U, V, ) = dU + P dV + N d() =
dV +
dV +
d()
(102)
U V,
V U,
U,V
which leads to the Maxwell relation,


()


=
U,V

N
U


(103)
V,

Using the chain rule on the left hand side yields,











1
T
T

=
= 2
() U,V
T U,V () U,V
T
() U,V

(104)

which is the first relation of Eq. (101). The second relation of equation (101) has T, V, N as independent variables,
so we use the Helmholtz free energy,






F
F
F
dT +
dV +
dN
(105)
dF = SdT P dV + dN =
T V,N
V T,N
N T,V
leading to the Maxwell relation,


S
V


=
T,N

P
T


(106)
V,N

The expression on the left hand side may be rewritten using the chain rule,
 






S
S
V
V 2 S
=
=
T,N
V T,N T,N
N V T,N

(107)

Equation (106) is then,


N2
2
V

(S/N )
V


=
T,N

(S/N )
V


=
T,N

P
T


=
V,N

P
T


(108)
,N

which is the second relation in Eq. (101)

Problem 19. From the second result of problem 18 show that








P
P
P
NT
=
+[
]2 2
S/N,N
T,N
T ,N
CV
Solution: We use the relation for a non-natural derivative,








P
P
P
T
=
+
S/N,N
T,N
T ,N S/N,N

(109)

(110)

15
Using the triple product rule, we have,







(S/N )

= 1
T S/N,N
T
(S/N ) T,N
,N

(111)

This yields,



=
S/N,N

T
(S/N )


,N

(S/N )


T,N

NT 1
=
CV 2

P
T


(112)
,N

Using the second expression in (101) and (112) in (110) yields the desired result.

Problem 20. Consider a system with an order parameter, x. If the volume(V ), energy(E) and particle number(N )
are fixed, then the system will choose the value of x with the maximum entropy. Now consider that the same system
is connected to a reservoir that has temperature T and chemical potential , so it can exchange energy and particles
with the reservoir. Show that the total entropy, that is the sum of the entropy of the reservoir and the system, is
maximized when


P
=0
(113)
x T,,V
Hint: You can assume the system and reservoir are at the same temperature and chemical potential as required by
equilibrium. Moreover you may use P V = T S E + N .
Solution: Since the variables that are kept fixed are T, , V , we use the grand potential G = U T S N , so
that,
dG (T, V, ) = SdT + P dV N d = d(P V )

(114)

Now we take a derivative of this expression with respect to x at fixed T, V, . Since the entropy is maximized, the
Grand potential is minimized, so that




P
G
= 0 = V
(115)
x T,V,
x T,V,

Problem 21. Consider a system where the thermodynamics are a function of temperature and another variable x
which can be varied by an experimentalist. Assuming that


E
= T ; and using F = E T S
(116)
S x
show that,


E
x


=
S

F
x


(117)
T

Solution: We take a derivative of the equation F = E T S with respect to x at constant T , to find,






 
F
E
S
=
T
x T
x T
x T
Using the formula for a non-natural derivative, we have,


  



E
E
S
E
=
+
x T
S x x T
x S
Using this result, along with the first of Eq. (116), in Eq. (118) proves Eq. (117).

(118)

(119)

16

Problem 22. Show that



CV = T

2F
T 2


(120)
N,V

Solution: The specific heat at constant volume is given by,




S
CV = T
T V,N

(121)

The Helmholtz free energy is given by,



F = U T S;

so

dF = SdT P dV + dN =

F
T


dT +

V,N

F
V


dV +
T,N

F
N


dN

(122)

T,V

so that,

S=

F
T


;

and

V,N

S
T


=
V,N

2F
T 2


(123)
V,N

Multiplying by T and using Eq. (121) proves the relation (120).

Problem 23. Show that



U=

(F/T )
(1/T )


(124)
N,V

Solution: Defining = 1/T , we have,








F
F
(F )
=F +
=F T

N,V
T N,V
N,V
where the last expression on the right hand side is found using the chain rule. We also have,






F
F
F
dF = SdT P dV + dN =
dT +
dV +
dN
T V,N
V T,N
N T,V

(125)

(126)

so that


F
T


= S.

(127)

V,N

Therefore,


(F )


= F + TS = U

(128)

N,V

as posed in Eq. (124).

Problem 24. Show that



V

P
T


= S;
,N

and V


=N
T,N

(129)

17
Now express the pressure of the ideal classical gas in terms of the variables and T and using this expression verify
the equations above.
Solution: We use the fact that the Gibbs free energy is N to write





G
=N
P T,N
P T,N

(130)

We also have,

dG =

G
T


dT +
P,N

G
P


dP +

T,N

G
N


dN = SdT + V dP + dN

(131)

T,P

so that


G
P


= V.

(132)

T,N

Using this relation in Eq. (130) proves the second identity in (129). To prove the first relation in (129), we use the
triple product identity,







T
P
= 1.
(133)
T ,N P T,N P,N
From Eq. (134) and G = N , we have,

S =

G
T


=N
P,N


(134)
P,N

Using the second relation in Eq. (129) for the second term in the product (133) and using (134) leads to the first
relation in (129).

Problem 25. From the fundmental thermodynamic relation show that






S
V
=
P T,
T P,

(135)

Solution: The fundamental thermodynamic relation is dU = T dS P dV + dN , so the independent variables are


S, V, N . To find the desired expression using a Maxwell relation approach, we need independent variables T, P, , so
we define Y = U T S + P V N , so that,






Y
Y
Y
dY = SdT + V dP N d =
dT +
dP +
d
(136)
T P,
P T,
T,P
so that,


Y
T


= S;
P,

Y
P


=V;

T,

= N

(137)

T,P

and hence,

Y
T


=

P,

T,

S
P


T,

Y
P


=

T,

P,

V
T


(138)
P,

Since the order of differentiation does not matter, as dY is an exact derivative, the relation (135) is proven.

18
Problem 26. Consider a single harmonic oscillator with energies n = h
(n + 1/2). Find the average energy of the
system as a function of temperature. Find the average occupancy of each energy level as a function of temperature.
Is there are regime where your result for the average energy is in agreement with the equipartition theorem?
Solution The energy levels of the Harmonic oscillator are given by n = h(n + 1/2), so the canonical partition
function is given by,
Z1 =

eh/2
1 eh

(139)

lnZ
1
h
= h + h

2
e
1

(140)

eh(n+1/2) =

n=0

so the average energy is,


< E >=
This can be written as,
< E >= h
(< n > +1/2);

where

< n >=

1
eh

(141)

In the high temperature limit, this reduces to < E >= 21 h


+ kB T which has the equipartition term (2 degrees of
freedom) plus a quantum correction that is negligible at high temperature.

Problem 27. For a pure state show that


2 =

(142)

(
)nm = ank amk

(143)

Also show that for a mixed state this identity fails.


Solution For a pure state,

and
(
2 )nm =

an al al am = ank amk

(144)

where we used
are given by,

al al = 1 for an orthonormal wavefunction. For a mixed state the elements of the density matrix
(
)nm =

pk ank amk

(145)

so the elements of the square of the density matrix are,


XX
(
2 )nm =
pk pk0 ank alk alk0 amk0
kk0

(146)

which is clearly not the same. The purity of the mixed state is,
XX
Pu = tr(
2 ) =
pk pk0 ank alk alk0 ank0
kk0

ln

(147)

19
Problem 28. Consider two single particle pure states with wavefunctions
| > =

1
(|0 > |1 >)
21/2

(148)

Write down the density matrix for each of these pure states and verify that they have purity equal to one. Now
consider the density matrix of a mixed state consisting of = p+ + + p , where p+ + p = 1. Find the purity of
this state, and the value of the mixing probability p+ at which the purity is smallest.
1
=
2
+

(
+ ) 2 =

1
4


1 1
1 1

1
=
2

(
)2 =

1
4


2 2
2 2

1 1
1 1


2 2
2 2

(149)

(150)

which confirms the identity. The purity is clearly Pu = 1. For the mixed case we have,
=

p+ + p
+

2
2

(151)

From which we find,


(
)2 = (

p+ + p 2
+
) = (p+ )2 + + (p )2 = (p+ )2 + (1 p+ )2 = 1 2p+ + 2p2+ 1
2
2

(152)

the minimum occurs at p+ = 1/2 and the maximum at p+ = 1.


Problem 29. Show that Eqs. (179) of the lecture notes reduce to Eqs. (178) in the eigenvalue basis.
Solution We use the relation
E
H
U
=
U

(153)

where U is a unitary matrix and E is the diagonal matrix with the eigenvalues of the hamiltonian H on the diagonal.
We then have,
X

) = tr(e U E U ) = tr(e H )
e E U
(154)
Z=
eEi = tr(e E ) = tr(U
i

U
=U
U
=
where we used the trace property tr(ABC) = T r(BCA) and the property of unitary matrices U
1, where

1 is the identity matrix. The other expressions are transformed in a similar way.

Solutions to problems for Part 2


Sample Quiz Problems
Quiz Problem 1. Write down the equation for the thermal de Broglie wavelength. Explain its importance in the
study of classical and quantum gases.
Solution
=

h
2mkB T

(1)

This is of the form h/pT , where pT = (2mkB T )1/2 is an average thermal momentum. Define the average interparticle
spacing of a gas Lc = (V /N )1/3 . If > Lc quantum effects become important in the thermodynamics.

Quiz Problem 2. Why are the factors 1/N ! and 1/h3N introduced into the derivation of the partition function of
the ideal classical gas?
Solution
The factor 1/N ! is needed to account for the fact that when an intergration is carried out over all phase space for
N particles, all permutations of the particle identities is included. For indentical particles this must be removed. The
factor 1/h3N takes account of the Heisenberg uncertainty principle which states that the smallest phase space volume
that makes sense is (
h/2)3 . The fact that it is 1/h3 instead of 1/(h/2)3 for each particle is to reproduce the high
temperature behavior of quantum gases.

Quiz Problem 3. Explain why the heat capacity at constant volume of a nitrogen molecule is roughly 1.5N kB at
1K but 2.5N kB at room temperature. What are the expressions for the heat capacity at constant pressure for these
two cases?
Solution
At low temperatures only the translational degrees of freedom are active so we have kB T /2 of energy for each
of three degrees of freedom. The specific heat is then 3kB T N/2. Once the temperature is larger than the spacing
between rotational energy levels h
2 /2I where I is the moment of inertia, then the rotational degrees of freedom are
also active as we get two more degrees of freedom. Only two rotational degrees of freedom contribute as the moment
of inertia about the molecule axis is very small, so the level spacing is large and this mode is not active at room
temperature. The heat capacity at constant pressure is CP = CV + N kB T .

Quiz Problem 4. In modeling the earths atmosphere using the equilibrium isothermal model, the gas pressure
decreases exponentially with altitude. Explain how this is reconciled with the equilibrium condition of constant
pressure found in our discussion of thermodynamics in Part 1 of the course.
Solution
The condition that the pressure of a thermodynamic system such as a gas must be a constant applies when there
are no externally applied potentials in the system. An external potential such as a gravitational field or a harmonic
trapping potential as occurs in atom traps leads to a force and this force is balanced by a pressure gradient in the
gas. This leads to the condition of hydrostatic equilibrium like that as used in the analysis earths atmosphere.

2
Quiz Problem 5. Discuss the assumptions used in the isothermal and adiabatic models of the atmosphere.
Solution
The isothermal model assumes that the atmostphere is at constant temperature which requires that thermal equilibrium has been established in all regions of the atmosphere. The adiabatic model assumes that no heat flow occurs
in the gas and is justified by the argument that the thermal conductivity of air is very low. These models are clearly
limiting cases with the truth being somewhere between the two. Surprisingly the adiabatic model seems to fit the
behavior of the troposphere quite well.

Quiz Problem 6. What is the Curie law? Derive this law for the spin half Ising paramagnet.
Solution
The spin half Ising paramagnet has Hamiltonian,
H = s h

Si

(2)

where Si = 1. The partition function and magnetization are then,


ZN = (2cosh(s h))N ;

m=

(ln(ZN ))
= N s tanh(s h)
(h)

(3)

The magnetic susceptibility is then,


=

m
N 2s
= N 2s sech2 (s h))
h
kB T

(4)

which shows that in the limit h 0, the susceptibility behaves as C/T which is the Curie law.

Quiz Problem 7. Paramagnets and also the Ising model in one dimension exhibit a peak in their specific heat,
with the low temperature behavior being an approach to zero exponentially and at high temperature an approach to
zero as 1/T 2 . Why is there a peak in the specific heat in these models. What cant we use the rule kB T /2 per degree
of freedom to find the specific heat?
Solution
A peak in the heat capacity usually occurs in systems where there is a gap between the ground state energy and
the first excited state, though this is not the only mechanism for a peak in CV . If the gap is , the the peak in
the specific heat occurs at roughly kB Tc , though the precise location depends on the system. The peak occurs
because at the temperature the first excited state begins to be populated leading to a rapid change in the internal
energy with temperature and this leads to an increase in the heat capacity

Quiz Problem 8. Explain the meaning of negative temperature in the Ising paramagnetic and how it can be used
to make a magnetic cooling device.
Solution Negative temperature occurs when S/E is negative. For the Ising paramagnet this condition applies
in the energy regime E > 0 as the peak in the many body density of states is at E = 0. Magnetic cooling can
be produced by preparing a state in a polarized spin configuration followed by removing the field. Once the field
is removed, the system becomes more disordered and in doing so absorbs heat from its surroundings. This leads to
cooling, just as gas cooling is produced when a gas expands as in conventional gas cycle refridgeration.

3
Quiz Problem 9. What is the meaning of the terms spontaneous symmetry breaking (or spontaneous magnetization) and breaking of ergodicity in the context of the two dimensional Ising model.
Solution In a ferromagnet there are two ground states: all up spins; or all down spins. At low temperatures, these
two states are separated by a large energy barrier that is impossible to overcome at zero applied field. This system
is non-ergodic as phase space is separated into to regions that are not connnected by any trajectories. Spontaneous
symmetry breaking refers to the fact that the system chooses between to states (either all up or all down) and so the
up down symmetry of the system is broken without the application of a symmetry breaking field.

Quiz Problem 10. State and give a physical explanation of the behavior of the chemical potential and the
fugacity z = e as temperature T 0, for both the non-relativistic and ultra-relativistic Bose gas.
Solution.
For the Bose gas as temperature goes to zero, the internal energy contribution dominates. As temperature goes to
zero all of the particles that are added go into the ground state, so the chemical potential goes to the ground state
energy. For the ideal gas case the ground state energy is zero, so the chemical potential goes to zero. The fugacity
therefore goes to one. This is true for both non-relativistic and ultra-relativistic gases. In the Bose condensed phase
the fugacity remains at one up to the BEC critical temperature.

Quiz Problem 11. Write down the starting expression in the derivation of the grand partition function, B for
the ideal Bose gas, for a general set of energy levels l , with degeneracy gl . Carry out the sums over the energy level
occupancies, nl and hence write down an expression for ln(B ).
Solution
For the case of Bose statistics the possibilities are nl = 0, 1, 2... so we find
!gl
gl Y
gl
M 
M 
Y X
Y
1
1
(l )nl
B =
e
=
=
1 zel
1 e(l )
n
l
l=1
l=1

(5)

where the sums are carried out by using the formula for a geometric progression. We thus find,
ln(B ) =

M
X

gl ln 1 zel

(6)

l=1

< nl >=

1
zel
[ln(B )] =
l
1 zel

(7)

Quiz Problem 12. In the condensed phase, superfluids are often described in terms of a two fluid model. Based
on the analysis of the ideal Bose gas, explain the physical basis of the two fluid model.
Solution
The two fluid model considers that the Bose condensed phase is a superfluid while the particles in the excited states
behave as a normal fluid. The normal fluid exhibits dissipation and viscosity, while the superfluid has very low values
of viscosity and other remarkable properties such as phase coherence.
Quiz Problem 13. Why is the chemical potential of photons in a box, and also acoustic phonons in a crystal,
taken to be zero?
Solution.

4
The lowest energy state of these systems is zero so any additional photons or phonons may be placed in this state.
This is explained by the fact that photons and phonons are massless bosons that can be created and destroyed, or
by arguing that we in essence have a Bose condensed phase of zero energy particles so = 0. A more subtle and
ultimately the full explanation is through an understanding of the interactions with the reservoir. In the case of
massive particles the reservoir contains a very large number of the same massive particles so the exchange with the
reservoir is through exchange of the same type of particle. In a photon or phonon gas, the reservoir is a system
of atoms where the photons or phonons may be absorbed and re-emitted as combinations of different photons or
phonons. For this reason the same amount of total free energy in the phonon or photon gas may be divided amongst
an arbitrary number of particles, so the chemical potential to add another particle must be zero.

Quiz Problem 14. Derive or write down the blackbody energy density spectrum in three dimensions.
Solution. The blackbody energy density spectrum follows from the equation for the energy of the photon gas in
three dimensions,
Z
Z
L 3
L
ehkc
eh
=
2(
(8)
U = 2( )3
dk 4k 2 (
)
d 4 2 (
hkc
h)

h
kc
2
1e
2c
1 eh
0
0
where we used = kc. we then write,
Z
U
=
d u(),
V

where,

u() =

h
3
2 c3 eh 1

Quiz Problem 15. Write down and explain the relationship between the intensity of radiation emitted by a
blackbody (Stefan-Boltzmann law) and the energy density of a photon gas in the blackbody.
Solution. The relationship between the intensity and the energy density of blackbody radiation is,
cU
= T 4
(10)
4V
The factor c/4 is explained as follows: The factor of c converts the energy density of an EM wave into the intensity
of radiation crossing a surface whose surface normal is in the same direction as the direction of wave propagation.
The factor of 1/4 has two pieces. First we note that emission from the surface of a blackbody is isotropic so half of
the radiation is emitted back into the blackbody. Moreover, the amount of radiation emitted to the exterior is also in
all directions on a hemisphere. To find the radiation emitted in the normal direction, we take the component of the
electric field in the normal direction, leading to a factor of cos(). However the intensity is the square of the electric
field, so it comes with a factor of cos2 (). The average of cos2 () leads to the second factor of 1/2.
I=

Quiz Problem 16. Explain the physical origin of the cosmic microwave background (CMB) blackbody spectrum
of the universe. It is currently at a temperature of TCM B = 2.713K. If the universe is expanding at a constant rate
L(t) = H0 t, where H0 is a constant what is the expected behavior of the temperature TCM B (t).
Solution. During the photon epoque of the early universe that is believed to have existed during the period from
10 seconds after the big bang to 377 thousand years after the big bang (that is believed to have occured roughly 13.7
billions years ago), the universe consisted of a gas of charged particles and photons that was equilibrated. At around
380 thousand years after the big bang, Hydrogen and Helium began to form, reducing the scattering of photons and
the universe became transparent. The cosmic microwave background is a remnant of the photon gas that existed
380 thousand years ago. Assuming that the photon gas making up the CMB has not changed significantly due to
scattering since that time, we can relate the temperature of the CMB to the size of the universe by assuming that the
energy in the photon gas is conserved, so that,
U = constant = L(t)3
where L(t) is the size of the universe. We then find T L3/4 .

4
2 kB
4
3 3T
15h c

(11)

Quiz Problem 17. Derive or write down the spectral energy density for blackbody radiation in a universe with
two spatial dimensions.
Solution. The blackbody energy density spectrum follows from the equation for the energy of the photon gas in
two dimensions,
Z
Z
eh
L 2
2
d 2(h)
=
L
d u()
(12)
)
U = 2(
2c
1 eh
0
Note that here I kept the two polarizations of light even though one of them is along the third direction. We then
have,
u() =

h
2
c2 eh 1

Quiz Problem 18. Derive or write down the Debye theory for the internal energy for phonons in a square lattice.
Derive the low and high temperature limits of the internal energy and specific heat for this system.
Solution. The energy density for the Debye model for the case of a square lattice comes from assuming that
phonons are an ideal Bose gas where there is one acoustic mode per atom. The chemical potential is taken to be zero,
so we have,
Z
L 2 kD
ehkvs
U = 2( )
(14)
dk 2k(hkvs )
2
1 ehkvs
0
where,
L
NA = ( )2
2

kD


2k dk,

so that

kD =

4NA
L2

1/2
(15)

The factor of two in the front of the energy equation takes into account the fact that there are two phonon modes
for the square lattice. This is a rough approximation as only one of the the two modes has the dispersion relation
p = pvs .
We define x = pvs , and kB TD = h
kD c leading to,
Z
T 2 TD /T
U
x2
=
4T
(
(16)
)
dx
L2
TD
ex 1
0
In the low temperature and high temperature limits this reduces to,
CV
T 2
(
) ,
N kB
TD

T << TD ;

CV = 2N kB

T >> TD

(17)

Quiz Problem 19. On a graph, illustrate the behavior of the chemical potential and the fugacity z = e of
Bose, Fermi and classical gases as a function of temperature. What is the chemical potential and fugacity of the the
Fermi and Bose gases as temperature T and as T 0.
Solution
At low temperatures the chemical potential of quantum gases approaches the mean of the energy of the highest
occupied and lowest unoccupied energy levels. For Fermions with a negligible gap at the Fermi energy the chemical
potential at T = 0 is the same as the Fermi energy. For Bosons, all particles may be in the lowest energy level
and for gases this is close to zero for large volumes, so the chemical potential of the Bose gas is zero at T = 0,
V . At high temperatures the Bose and Fermi gases act like a classical gas so their chemical potential is given by,
= kB T ln(N 3 /V ) to leading order. The fugacity of all three gases approaches zero at sufficiently high T , while at
low T the fugacity of the Bose gas approaches one while that of the Fermi gas diverges.

Quiz Problem 20. Write down the starting expression in the derivation of the grand partition function, F for
the ideal Fermi gas, for a general set of energy levels l . Carry out the sums over the energy level occupancies, nl and
hence write down an expression for ln(F ).
Solution
F =

X
n1

...

PM
l=1

(l )nl

nM

M 
Y

M
 Y

1 + e(l ) =
1 + zel

(18)

l=1

l=1

where z = e and each sum is over the possiblities nl = 0, 1 as required for Fermi statistics. We thus find,
ln(F ) =

M
X

ln 1 + zel

(19)

l=1

Quiz Problem 21. White dwarf stars are stable due to electron degeneracy pressure. Explain the physical origin
of this pressure.
Solution
Even in the ground state, the internal energy of the Fermi gas is positive. This is due to the fact that only one
Fermion can be in each energy level so high energy states are occupied at zero temperature. As the density increase,
the Fermi energy or energy of the highest occupied state, increases. The pressure is the rate of change of the energy
with volume so the pressure increases with the density. This degeneracy pressure opposes gravitational collapse
and stabilizes white dwarf stars.

Quiz Problem 15. Explain the physical origins of the paramagnetic and diamagnetic contributions to the magnetization of the free electron gas.
Solution. The paramagnetic contribution to the magnetization of the free electron gas is the change in the spin
polarization due to the application of a magnetic field. The diamagnetic contribution to the magnetization is due to
changes in the electron orbitals due to the application of a magnetic field. The diamagnetic contribution can occur
even if there is no net spin. To a first approximation, we can add the paramagnetic and diamagnetic contributions.
When a paramagnetic contribution occurs, these two contributions are usually of opposite sign.

Assigned problems
Assigned Problem 1. From the density of states for an ideal monatomic gas (E) given in Eq. (26) of the notes,
find the Sackur-Tetrode equation for the entropy, Eq. (27) of the notes.
Solution.

(E) =

2 1/2 V N (2mE)3N/21/2
N !h3N
( 3N
2 1)!

(20)

Using Stirlings approximation and dropping constants, we have


"
#

3/2
V
3
3N
3 3
V 4mU
5
kB ln((E)) = kB N [ln( 3 ) ln(N ) + 1 ln(
) + + ln(2mU )] = N kB ln[
]+
h
2
2
2 2
N 3N h2
2

(21)

Assigned Problem 2. Using the canonical partition function for the ideal gas, show that,
(E)2 = kB T 2 Cv

(22)

Solution.
From Part 1 page 28, and using the ideal classical gas expression ZN = V N /(N !3N ), we have,
E 2 =

2
V N (2m)3N/2
3N
2 ln(Z)
3
= E 2 =
[ln(

ln()] = N (kB T )2 = kB T 2 CV
2
2

N !h3N
2
2

(23)

where CV = 3N kB /2 for the classical monatomic non-relativistic ideal gas in three dimensions.

Assigned Problem 3. Using the grand partition function of the ideal classical gas show that,
(N )2 = N kB T T

(24)

Solution. From Part 1 page 28, and using the expression for the grand partition function for the classical gas we
have,
(N )2 = (kB T )2

2 ln()
V
PV
V
= (kB T )2 3 2 e = 3 e =
=N
2

kB T

(25)

where we used (II.25) to write z = ln() = P V /kB T . Also the right hand side of Eq. (17) for the classical ideal gas
is,
N kB T T = kB T

N2
N2
T = kB T
=N
V
PV

(26)

where we used Eq. (II.17) for T for the ideal gas

Assigned Problem 4
Consider a gas of N atoms in volume V . Each atom has one unpaired electron in its outer energy level that is a zero
angular momentum state, i.e. the ground state is l = 0 and has an unpaired electron. A magnetic field is applied to
this gas. Write down the canonical and grand canonical partition functions of this system taking into account center of
mass and spin degrees of freedom, assuming that the magnetic interaction is paramagnetic with no spin-orbit coupling.
Solution The translational and spin degrees of freedom do not interact, so the canonical partition function is
simply a product of the two subsystem canonical partition functions,
ZN =

VN
(2cosh(s h))N
N !3N

(27)

The grand partition function is,


=

z N ZN = ez

where

V
2cosh(s h)
3

(28)

Assigned Problem 5.
Consider a gas where each atom can have one of two energy levels 0 and 1 . Find the average energy per particle
taking into account the ideal monatomic gas behavior and the two internal energy levels. Find the specific heat of
the system in the limits T 0 and T .

8
Solution The canonical partition function is again a product of the two subsystem partitions functions so that,
ZN =

VN
(e0 + e1 )N
N !3N

(29)

so the internal energy is,


U = Un + Ui =

0 e0 + 1 e1
3
N kB T + N
2
e0 + e1

(30)

where Un is the contribution from the nuclear or translational degrees of freedom while Ui is the contribution from
the internal degrees of freedom.

Assigned Problem 6.
Consider a dissociating system at thermal equilibrium, where,
A B + B

(31)

Ignore the internal degrees of freedom of both species and take the mass of an A particle to be twice that of a B
particle. Given that N = 2NA + NB is fixed, that the volume is fixed, and that the binding energy of A is , find the
temperature dependence of NA and NB and find a simple expression for the ratio < NB >2 / < NA >.
Solution
In this system there is an exchange of particles between the subsystem consisting of B atoms and the subsystem A
consisting of diatomic molecules with 2 B atoms. If there are NA molecules and NB free atoms the canonical partition
functions for the two subsystems is,
ZA =

V NA
eNA ;
A
NA !3N
A

ZB =

V NB
B
NB !3N
B

(32)

where  is the binding energy of the molecule. The equilibrium condition is that the chemical potential of an atom of
B must be the same whether it is bound in a molecule or free. The chemical potential of a free B atom is,
B =

(kB T lnZB ) = kB T ln(


)
NB
NB 3B

(33)

while the chemical potential of an A dimer consisting of two bound B atoms is,
A =

V
)
(kB T lnZA ) = kB T ln(
NA
NA 3A

(34)

At equilibrium a B atom has the same chemical potential whether it is free or bound into an A dimer. We then have
the condition,
B =

1
A
2

(35)

and using this condition we find the relation,


NB2
V
= 3 e
NA
8A

(36)

where we used mA = 2mB so that 2B = 22A . Using this relation in combination with the condition N = 2NA + NB ,
we can find the behavior of NB as a function of temperature by solving,
2NB2
V
= 3 e = x
N NB
8A

(37)

so that,
2NB2 + xNB xN = 0;

so

NB =

p
p
x
1
[x x2 + 8N x] = [1 1 + 8N/x]
4
4

(38)

9
The grand partition function may also be used to find the ratio < NB >2 / < NA >, by realizing that = A B
and using,
X
X
NB
NA
zB
ZB (N ) = Exp[zB V /3B ]; A =
zA
ZA (N ) = Exp[zA V /3B ]
B =
(39)
NB

NA

The average number of atoms of each type is given by,


ln(B )
V
= eB 3
(B )
B

(40)

V
ln(A )
= eA e 3
(A )
A

(41)

< NB >=
and
< NA >=

Now use the fact that A = 2B and take the ratio < NB >2 / < NA > to reproduce the result found earlier.

Assigned Problem 7.
Calculate the specific heat of the model defined by the Hamiltonian Eq. (51) of the text. Is there a peak in the
specific heat? Why or why not?
Solution The internal energy per spin of this model is hm so that,
1
]
s h

U = s hN [coth(s h)

(42)

Defining x = s h, we find the specific heat to be


Ch = s hkB N [

1
1
]

x2
sinh2 (x)

(43)

This increases monotonically with temperature and has no peak.

Assigned Problem 8.
Generalize the transfer matrix method for the one dimensional Ising model to the case of an applied field (i.e.
combine the Hamiltonians in Eq. (41) and (55)) and calculate the magnetic susceptibility at low field. Does it obey
a Curie law?
Solution It is convenient to write the Hamiltonian in the form,
H = J

Si Si+1 s h

X1
i

(Si + Si+1 )

(44)

as this makes the transfer matrix more symmetric. We then find,


T++ = eJ+s h ;

T+ = eJ = T+ ;

T = eJs h

(45)

and we seek the largest eigenvalue of this matrix. Defining = eJ ; = es h the characteristic equation for T is,
2 ( +

1
1
) + 2 2 = 2 2eJ cosh(s h) + 2sinh(2J)

(46)

which has solutions,


=

1 J
[2e cosh(s h)
2

4e2J cosh2 (s h) 8sinh(2J)]

(47)

10
which simplifies to,
= eJ [cosh(s h)

sinh2 (s h) + e4J ]

The eigenvalue with the plus sign dominates, so we find the magnetization using,


q
M
1

J
2
4J
m=
]
=
ln(Z) =
ln e [cosh(s h) + sinh (s h) + e
N
N (h)
(h)

(48)

(49)

This simplifies to,


m(h, T ) = s

Sinh(hs )
2s he2J ;
+ e4J ]1/2

[Sinh2 (s h)

1d Ising.

(50)

where the latter from is correct in the small h limit. The magnetic susceptibility at small field is then
=

m
2
s e2J
h
kB T

as h 0

(51)

This diverges exponentially at low T due to the incipient phase transition in the 1D Ising model.

Assigned Problem 9.
Consider the Peierls argument for the two dimensional spin half Ising model on a square lattice, where the topological
excitation is a flat domain wall dividing a domain of up spins and down spins. Does this argument suggest that finite
temperature spontaneous symmetry breaking is possible in the two dimensional Ising model?
Solution
In the two dimensional Ising model the Peierls estimate of the difference in free energy between the ground state
and a state with one domain wall in it is,
F = 2JL kB T ln(L).

(52)

Clearly the energy cost is larger than the entropy gain so the ordered state is stable.

Assigned Problem 10. Using the energy levels of a particle in a box and MB statistics to fill the energy levels,
derive the classical gas thermodynamics.
Solution
The grand partition function is given by,
Y
M B =
Exp[gi e(l ) ] so

ln(M B ) =

gi e(l )

(53)

Using the energy levels of a particle in a box and taking the continuum limit and setting gi = 1, this becomes,
Z
2
L 3
4k 2 zep /(2m)
(54)
ln(M B ) = ( )
2
0
Using cartesian co-ordinates we find,
L
ln(M B ) = z( )3
h

Z

p2 /(2m)

3


= zV

2m

3/2
(55)

so that,
ln(M B ) = z
as for the classical gas.

V
3

(56)

11
Assigned Problem 11. In lectures we showed that at high temperatures the equation of state of the Bose gas
reduces to the ideal classical gases. Derive the next term in the expansion of the equation of state of the ideal Bose
at high temperatures.
Solution. For the Bose case, expanding to second order gives
z2
N 3
z2
= g3/2 (z) = z + z1 + 0
V
2 2
2 2

(57)

where z0 = N 3 /V is the leading order solution, we then find,


z2
z1 = z0 0
2 2

(58)

We substitute this into the second order expansion of the equation of state,
z02

z0 22 )
z2
1
z2
1
z2
P
1

+ ...]
= 3 [z + + ...] 3 [z1 + 1 + ...] 3 [z0 0 + (
kB T

4 2
4 2
2 2
4 2

(59)

keeping terms to order z02 gives,


P
z0
1
1
= 3 [1 z0 ( ) + ...]
kB T

2 2 4 2

(60)

substituting z0 = N 3 /V gives,
PV
1 3 N
=1
+ ....
N kB T
4 2 V

Bose gas

(61)

The pressure is lower than the classical gas due to the effective attraction of Bosons.

Assigned Problem 12. Derive expressions for , P V , N/V and U for the non-relativistic Bose gas in one and
two dimensions. White general expressions that are valid in any dimension. Find the leading order terms in the high
temperature expansions for these quantities. Discuss the behavior of the non-relativistic Bose gas at low temperatures
in one and two dimensions. Is a finite temperature Bose condensation predicted ? Explain your reasoning.
Solution. For a Bose gas with dispersion relation p = p2 /2m in d dimensions
P
1
1
= d gd/2+1 (z) d ln(1 z);
kB T

N
1
1 z
= d gd/2 (z) + d
;
d
L

L 1z

U
d kB T
=
gd/2+1 (z)
d
L
2 d

(62)

These results are found from the integral forms in one and two dimensions below along with the three dimensional
result derived in lectures.
Z
2
L
ln(B ) =
2dpln(1 zep /2m ) ln(1 z); 1 dimension
(63)
2
h 0
and
ln(B ) = (

L 2
)
2
h

2pdpln(1 zep

/2m

) ln(1 z)

2 dimension

(64)

A series expansion as carried out for these integrals and the similar forms for N/V and U lead to the results above.
To leading order in the fugacity of the equation for N/V , we find the chemical potential to be the same as that of the
ideal classical gas in d dimensions, i.e. z = N (/L)d . The dimensional dependence comes from the different powers
of the factor (L/h)d , and the factors of p in the integral. The integrals that are needed are,
Z
Z
2
1
1
x2 l
xe
dx = ;
ex l dx = ( )1/2
(65)
2l
2 l
0
0

12
We then have,

L 2m 1/2 X z l
PL
= ln(B ) =
(
)
kB T
2
h
l

2dxex

ln(1 z);

1 dimension

(66)

l=1

and

P L2
L 2 2m X z l
= ln(B ) = (
) (
)
kB T
2
h

l
l=1

2xdxex

ln(1 z);

2 dimensions

(67)

which reduce to the expression given above for P/(kB T ).


At at any temperature, the chemical potential potential of the ideal non-relativistic Bose gas in dimensions less
than 2 + cannot be one in the thermodynamic limit, as,
gn (z) =

X zl
ln

(68)

diverges for z = 1 and n 1. Since z cannot approach one, the term z/(V (1 z)) approaches zero in the thermodynamic limit, indicating that it is impossible for a finite fraction of the particles to be in the ground state. There is
therefore no BEC phase transition at finite temperature in one and two dimensional ideal non-relativistic Bose gases.

Assigned Problem 13. For the 3-D non-relativistic case, find the entropy of the ideal Bose gas in the condensed
phase T < Tc . Compare to the classical gas.
Solution. Using the thermodynamic relation, U = T S P V + N , we find,
T S = U + P V N =

5
P V N
2

(69)

For the Bose gas at T < Tc where = 0, we have P/kB T = (5/2)/3 and using P V = 2U/3 we find,
TS =

5
5 kB T V
PV =
(5/2)
2
2 3

(70)

The entropy of the Bose gas in the condensed phase is thus proportional to T 3/2 , which goes to zero at low temperature.
The entropy of the classical gas is given by Eq. (16) of lecture notes part 2, so that,
V
5
)+ ]
(71)
N 3
2
The classical gas has higher entropy than the Bose gas in the condensed phase. In the condensed phase the number of
excited state particles grows at T 3/2 and only these particles contribute to the entropy of the Bose gas. The ground
state particles do not contribute to the entropy and as the temperature is reduced almost all particles are in the
ground state.
S = N kB [ln(

Assigned Problem 14. Show that a d dimensional Bose gas with dispersion relation p = cps obeys the
relation,
s U
d Ld
Solution. The equations for a Bose gas with this dispersion relation in d dimensions is written as,
Z
Z
s
s
cd
P
U
cd
zecp
d1
s
= d
dp pd1 ln(1 zecp );
=
dp
p
(cp
)
kB T
h 0
Ld
hd 0
1 zecps
P =

Integrating the pressure equation by parts gives,


Z
Z
s
s
s
1
s
zecp
d1
s
I1 =
dp pd1 ln(1 zecp ) = pd ln(1 zecp )|
+
dp
p
(cp
)
0
d
d 0
1 zecps
0

(72)

(73)

(74)

The first term on the right hand side is zero and comparison of the remaining integral with the energy equation proves
relation (63).

13

Assigned Problem 15. Find the thermodynamic properties, P V , U , S, CV , N of a photon gas in d dimensions.
Show that the entropy per photon is independent of temperature.
Solution. We use the relations,
 d
d Z

Z
L
1 d
xd1
epc
L
d1
cd p dp
=
2c
(
dx
)
N =2
d
2
h
1 epc
h
c
ex 1
0
0

(75)

and

U =2

L
2
h

d Z

cd pd1 dp (pc)

epc
= 2cd c
1 epc

 d
Z
L
1
xd
( )d+1
dx x
h
c
e 1
0

along with P V = sU/d, with s = 1 and the integral,


Z s1
x dx
= (s)(s),
ex 1
0

(76)

(77)

to find,
 d
kB
L
( )d+1 T d+1 ;
U = 2cd cd!(d + 1)
h
c

 d
L
kB
N = 2cd (d 1)!(d)
( )d T d
h
c

(78)

We also have,
T S = U + P V N = (d + 1)U/d T d+1

(79)

From this it is evident that both S and N are proportional to T d , so S/N is temperature independent. Also,
CV =

U
= 2(d + 1)!(d + 1)cd c
T

 d
L
kB
( )d+1 T d
h
c

(80)

Assigned Problem 16. Find the thermodynamic properties, U and CV for the Debye phonon model in d dimensions.
Solution. We use the relation,

 d Z pd
 d
Z
L
kB d+1 d+1 xD
epvs
L
xd
U =d
=
dc
v
(
)
T
cd pd1 dp (pvs )
dx
d
s
pv
x
s
2h
1e
h
vs
e 1
0
0

(81)

where the factor of d in front ensures that we recover the high temperature equipartition result. xD = pD vs and we
define the Debye temperature through kB TD = pD vs , so that xD = kB TD = TD /T . We also use the integral,
Z s1
x dx
= (s)(s),
(82)
ex 1
0
to find that at low temperatures TD /T , so the behavior is like that of the photon gas in d dimensions, with
c vs , and multiplied by d/2 due to the difference in degeneracy (2 for photons, d for phonons). We then have ,
 d
L
kB
U = dcd vs d!(d + 1)
( )d+1 T d+1 ;
h
vs

U
CV =
= d(d + 1)!(d + 1)cd vs
T

 d
L
kB
( )d+1 T d
h
vs

(83)

At high temperatures, the behavior is like that of a classical gas in a harmonic potential so that U = dN kB T, CV =
N dkB , and P V = 2U/d.

14

Assigned Problem 17. Carry through the analysis of the Bose gas for the case of massive ultrarelativistic particles
where 6= 0. What is the lower critical dimension for a BEC transition in this case?
Solution. For the ultrarelativistic Bose gas, we use, k = h
kc, so that,

N=

L
2
h

3 Z

4k 2 dk

z
zehkc
+
1 zehkc
1z

(84)

which reduces to,


1
V
N=
2 2 (
hc)3

dxx2 zex

z l exl +

l=0

z
.
1z

where we used the expansion 1/(1 x) = 1 + x + x2 + x3 .... Using the integral,


Z
Z
1 s1 y
(s 1)!
s1 lx
dxx e
= s
y e dy =
(s = 3)
l
ls
0
0

N=

V
1
2
2 (hc)3

dx

X
2z l

l=1

l3

(85)

2
l3

(86)

z
.
1z

(87)

Since for z = 1, the series is convergent, there is the possibility of Bose condensation at finite temperature and by
setting z = 1 we find the critical condition,
N=

1
V
(3)
2
(hc)3

(88)

A similar calculation shows that in two dimensions the ultrarelativistic ideal Bose gas has a phase transition at finite
temperature while in one dimension a finite temperature BEC transition does not occur.

Assigned Problem 18. At high temperatures we found that the ideal quantum gases reduce to the ideal classical
gases. Derive the next term in the expansion of the equation of state of the ideal Fermi gas at high temperatures,
and verify that,
PV
1 3 N
=1+
+ ....
N kB T
4 2 V

Fermi gas

(89)

The pressure of the ideal Fermi gas is higher than that of the classical gas at the same temperature and volume.
Why? Carry out a similar expansion for the Bose gas. Is the pressure higher or lower than the ideal classical gas at
the same values of T, V ? Why?
Solution. For the Bose case, expanding to second order gives
z2
N 3
z2
= g3/2 (z) = z + z1 + 0
V
2 2
2 2

(90)

where z0 = N 3 /V is the leading order solution, we then find,


z2
z1 = z0 0
2 2

(91)

We substitute this into the second order expansion of the equation of state,
z02

z0 22 )
P
1
z2
1
z2
1
z2

= 3 [z + + ...] 3 [z1 + 1 + ...] 3 [z0 0 + (


+ ...]
kB T

4 2
4 2
2 2
4 2

(92)

15
keeping terms to order z02 gives,
1
z0
1
P
= 3 [1 z0 ( ) + ...]
kB T

2 2 4 2

(93)

substituting z0 = N 3 /V gives,
PV
1 3 N
=1
+ ....
N kB T
4 2 V

Bose gas

(94)

Analysis for the Fermi gas is the same, except that the sign on the correction term is positive. The Bose gas has
reduced pressure as compared to the classical gas at the same temperature, while the Fermi gas has higher pressure
than the classical case. This expansion can be extended to higher order and in general is written as,

X
PV
=
al l1 ;
N kB T

where

l=1

3 N
V

(95)

This expansion is valid when is small, which means low density and/or high temperatures. In general expansions
of this type are called virial expansions and have played an important role in characterizing interactions in gases.
In classical gases the second virial coefficient a2 is determined by the strength of the pair interactions, as we shall
see in Part 3 of the course. Here the terms l > 1 are due to quantum effects. In real quantum gases, both quantum
effects and interactions can be important. Recall than our condition for quantum effects to be important was that the
interparticle spacing Lc < . When this is true is significant and more terms are required in the virial expansion.

Assigned Problem 19. By expanding the denominator of the integral, 1/(1 + y) for small y = zex show that,

1/2

f3/2 (z) =

dx x2

X (1)l+1 z l
zex
2 =
x
1 + ze
l3/2

(96)

l=1

Solution.
We use the expansion

1
= (1 y + y 2 y 3 ...);
1+y

and

2 ax2

x e

dx =

4a3/2

(97)

so that,
f3/2 (z) =

4
1/2

Z
0

dx x2

X
zex
(1)l+1 z l
2 =
x
1 + ze

l=1

dx x2 elx =

X
(1)l+1 z l
l=1

l3/2

(98)

Assigned Problem 20a. Derive expressions for Z, , P V , and U for the classical gas in one and two dimensions. Are the results what you expect? How do they compare with the result in three dimensions. White general
expressions that are valid in any dimension.
Solution In d-dimensions, the partition functions are,
Z=

LdN
,
dN N !

= ez

L
= ( )d

(99)

LdN
)
dN N !

(100)

F = kB T ln(Z) = kB T ln(

S=

F
T


= kB ln(
Ld ,N

LdN
d
) + N kB
dN
N!
2

(101)

16
The internal energy is found by combining (31) and (32), so that,
U = F + TS =

d
N kB T
2

(102)

The pressure is given by,



P =

F
Ld

N
kB N T
=
,
Ld
Ld

= kB T
T,N

(103)

which is the ideal gas law, while the chemical potential is,


F
=
= kB T ln(d N/Ld )
N T,Ld

(104)

Assigned Problem 20b. Derive expressions for , P V , N/V and U for the Fermi gas in one and two dimensions.
Write general expressions that are valid in any dimension. Find the leading order terms in the high temperature
expansions for these quantities. Are the results what you expect? How do they compare with the result in three
dimensions, and with the classical gas.
Solution The relations for the non-relativistic Fermi gas are,
P
1
= d fd/2+1 (z);
kB T

N
1
= d fd/2 (z);
Ld

U
d kB T
=
fd/2+1 (z)
Ld
2 d

(105)

These results are found from the integrals,


L
ln(F ) =
2
h

2dpln(1 + zep

/2m

1 dimension

);

(106)

and
ln(F ) = (

L 2
)
2
h

2pdpln(1 + zep

/2m

2 dimension

(107)

These integrals and the analogous equations for N and V are expanded as in the three dimensional case. Following
the procedure given in the solution to problem 7, we have,

L 2m 1/2 X (1)l+1 z l
PL
= ln(F ) =
(
)
kB T
2h
l

2dxex l ;

1 dimension

(108)

l=1

and

P L2
L 2 2m X (1)l+1 z l
= ln(F ) = (
) (
)
kB T
2
h

l=1

2xdxex l ;

2 dimensions

(109)

which reduce to the expression for P/kB T given in Eq. (42). The leading order expansion of fd/2 (z) at high
temperature gives the chemical potential of the ideal classical gas in d dimensions so we recover the classical gas in d
dimensions at sufficiently high temperatures.

Assigned Problem 21. Discuss the behavior of the Fermi gas at zero temperature in one and two dimensions. Is
there different behavior as a function of dimension? Explain your reasoning.
Solution. We calculate the energy and degeneracy pressure to see if there is a dependence on dimension. We only
carry out the ground state calculation. The Fermi wavevector in one, two and three dimensions is given by,
kF 1 =

N
;
L

kF 2 = (

4N 1/2
) ;
L2

kF 3 = (

6 2 N 1/3
)
V

(110)

17
so the Fermi energy is given by,
F 1 =

2
h
(N )2 ;
2mL2

F 2 =

2
h
(4N );
2mL2

F 2 =

2
h
(6 2 N )2/3 ;
2mL2

(111)

The internal energy is given by,


U =(

L d
)
2

dd k

h2 k 2
2m

(112)

In one two and three dimensions we find,


U1 =

1
N EF 1 ;
3

U2 =

1
N EF 2 ;
2

U3 =

3
N EF 3
5

(113)

The degeneracy pressure is given by,



P =

U
Ld


(114)
N,T

Since EF is proportional to 1/L2 doing the derivative with respect to L, L2 and L3 in one two and three dimensions,
leads to the following expressions for the degeneracy pressure,
P1 =

2
N EF 1 ;
3L

P2 =

1
N EF 2 ;
2L2

U3 =

2
N EF 3 .
5L3

(115)

For fixed number of particles, the degeneracy pressure and internal energy are much higher for the one and two
dimensional cases, first because EF grows much more rapidly with N as the dimension is reduced and second because
the prefactor grows more slowly with L as the dimension is reduced. This results are expected as particles are more
confined in one and two dimensions, so the effect of Pauli exchange is stronger, so the total energy is expected to
grow more rapidly in lower dimension and the degeneracy pressure should be higher.

Assigned Problem 22. Show that a d dimensional Fermi gas with dispersion relation p = cps obeys the
relation,
P =

s U
d Ld

Solution. The equations for a Fermi gas with this dispersion relation in d dimensions is written as,
Z
Z
s
s
P
cd
U
cd
zecp
d1
s
= d
dp pd1 ln(1 + zecp );
=
dp
p
(cp
)
kB T
h 0
Ld
hd 0
1 + zecps
Integrating the pressure equation by parts gives,
Z
Z
s
1 d
s
zecp
d1
cps
cps
d1
s
I1 =
dp p ln(1 + ze
) = p ln(1 + ze
)|0 +
dp p (cp )
d
d 0
1 + zecps
0

(116)

(117)

(118)

The first term on the right hand side is zero and comparison of the remaining integral with the energy equation proves
the relation.

Quiz Problem 23. Find the leading order term in the temperature dependence of the internal energy and specific
heat of an three dimensional ultrarelativistic Fermi gas at low temperature.
Solution. The equations for U and N for the three-dimensional ultra-relativistic Fermi gas are,
Z
Z
pc
x
L 3
4 L 3
2 ze
2 ze
dp p
dx
x
N = 4( )
=
(
)
h
1 + zepc
(c)3 h
1 + zex
0
0

(119)

18
and
L
U = 4c( )3
h

dp p3

4c L 3
zepc
=
( )
1 + zepc
(c)4 h

dx x3

zex
1 + zex

(120)

We may expand the integral at small z, but this is not useful at low temperature. Instead we carry out the Sommerfeld
expansion. Here we write it in more general form, generalizing Eq. (II.73) to,
Z
Z
Z
x
1
ex
1
s1 ze
s1
s
Is =
dx x
=
dx
x
dx
x
(121)
=
1 + zex
ex + 1
s 0
(ex + 1)2
0
0
Expanding xs about we have,
1
xs = ( + (x ))s = f (0) + (x )f 0 (0) + (x )2 f (0) + ....
2

(122)

where f (y) = ( + y)s , so that f (0) = s , f 0 (0) = s s1 , 00 (0) = s(s 1) s2 . so that


1
xs = ( + (x ))s = s + (x )s s1 + (x )2 s(s 1) s2 + ....
2

(123)

Following the procedure of Eq. (II.77) and (II.78), we then have,


Is =

1 s
1
[ I0 + s s1 I1 + s(s 1) s2 I2 + ...);
s
2

Z
where

In =

dt

tn et
+ 1)2

(et

(124)

I0 = 1, while by symmetry In is zero for odd n. For even n > 0, In is related to the Reimann zeta function, through,
In = 2n(1 21n )(n 1)!(n),

with

(2) =

2
4
, (4) =
,
6
90

(6) =

6
945

(125)

Since the odd integral I0 = 1, I1 = 0, I2 = 2 /3, we find,


Is =

1 s 2
[ +
s(s 1) s2 + ...]
s
6

(126)

Up to a prefactor that is defined differently here, the expansion above is consistent with Eqs. (II.78) and (II.88) as
they must be. The expansions we need are then,
N
4 1 3
=
[ + 2 + ...]
V
(hc)3 3

(127)

U
4c 1 4
= 3
[ + 2 2 2 + ...]
V
h (c)4 4

(128)

and

The leading order term in the expansion of the chemical potential is found using,
N
4 1
=
(0 )3
V
(hc)3 3


so

0 = hc

3N
4V

1/3
(129)

The next correction is found using,


03 = (1 )3 + 2 0 ;

so that

1 = 0 (1

2
2 kB T 2
)
=

[1

(
) + ...].
F
3(0 )2
3 F

(130)

where F = 0 = 0 /. The internal energy expansion is,


U
c
2 2 2
c
4 2 kB T 2
4
4
= 3

[1
+

+
...]


[1

(
) + ...][1 + 2 2 2
F
F + ...]
V
h (c)4
3
h3 (c)4
3
F

(131)

Solutions to problems for Part 3

Assigned problems and sample quiz problems


Sample Quiz Problems
Quiz Problem 1. Draw the phase diagram of the Ising Ferromagnet in an applied magnetic field. Indicate the
critical point. Plot the magnetization as a function of the applied field for three temperatures T < Tc , T = Tc , T > Tc .
Quiz Problem 2. Plot the behavior of the magnetization of the Ising ferromagnet as a function of the temperature,
for three applied field cases: h < 0, h = 0, h > 0. Indicate the critical point.
Quiz Problem 3. Write down the definition of the critical exponents , e , , , and . What values do these
exponents take within mean field theory.
Solution.
CV t ;

m t ;

t ;

m(Tc ) h1/ ;

c(r) er/ /rd2+

(1)

where = t , and t = |T Tc |. Within mean field theory = 0, = 1/2, = 1, = 3, = 0, = 1/2

Quiz Problem 4. Write down the mean field equation for the Ising ferromagnet in an applied field, on a lattice
with co-ordination number z and exchange constant J. From this equation find the critical exponent for the Ising
ferromagnet within mean field theory.
Solution.
1
m = T anh(Jzm + h) (Jzm + h) (Jzm + h)3
3

(2)

at the critical point Jz = 1, so m h1/3 and hence = 3.

Quiz Problem 5. Write down the scaling hypothesis for the magnetization, susceptibility, free energy and correlation function. From these relations, find the Fisher, Widom and Rushbrooke critical exponent relations. Also write
down the hyperscaling relation.
Solution
We assume that the correlation length is the key quantity in the scaling theory so that the scaling behavior is of
the form,
F (T, h) = t2 Fs (h y );

M (T, h) = t Ms (h y );

(T, h) = t s (h y );

C(r) = rp Cs (r/, h y )

(3)

where t = |T Tc |, and y > 0. We also define y = t , so that y = , where is the gap exponent. We also have
p = d 2 + , and = t . The scaling functions have the property that as their argument x = h y = h/t goes to
zero, the scaling functions must approach a constant. Moreover the scaling assumption states that for h < y the
scaling functions are constant. Moreover, as x , the scaling functions go to zero. First consider the behavior of
the magnetization when we are at the critical point, so that,
M (t = 0, h 6= 0) t Ms (x ) h1/ ;

so that

Ms (x) xk

(4)

(5)

where,
t xk = t (

h k
) = h1/ ;
t

so that

k = 1/;

and

2
Now consider the relation between the magnetization and the susceptibility,
t

dh t t t ;

so that

(6)

In a similar manner,
Z
F

M dh t t t2 ;

so that

+=2

(7)

Finally, consider the scaling of the correlation function in the case where h y is zero, so that Cs is a constant for r <
and zero otherwise. We then have,
Z

d rC(r)

drrd1 rp Cs (r/, h y ) d(d2+) t ;

so that

= (2 )

(8)

These exponent relations are usually written in the form,


= + ;

= (2 ) (F isher);

+ 2 + = 2

(Rushbrooke);

= ( 1)

(W idom)

(9)

Since we have added the gap exponent , there are seven exponents in the problem. We have four exponent relations
so that only three exponents are independent. Josephson introduced another relation, called the hyperscaling relation.
He introduced the hypothesis that the singular part of the free energy scales as 1/ d . This implies that,
fsing d t2 ;

so that

d = 2 (Josephson, or hyperscaling relation)

(10)

The hyperscaling relation is considered the most likely of the scaling relations to fail and for example is known to fail
in some heterogeneous models such as the Spin glass model.

Quiz Problem 6. Find the domain wall energy for the Ising (O(1) model) and for the O(2) model. From these
expressions find the lower critical dimension for these two problems.
Solution
We consider the ground state energy of a domain wall. The O(1) model is the Ising case so the spin is either
up or down, while O(2) corresponds to a spin vector that has unit length and can take any angle 0 < < 2. A
ferromagnetic interaction favors all the spins pointing in the same direction for both models. For both cases, a domain
wall created by setting the spins on one side of a hypercubic system in the up direction, while the spins on the opposite
size are in the down direction. The interface between the up and down domains that this boundary condition creates
is called the domain wall. For an Ising system we found that in the ground state,
Ising
EDW
= 2JLd1

(11)

For the O(2) case, the domain wall energy is,


continuous
EDW

d2
JL
2

(12)

The lower energy of domain walls in systems with an order parameter that can take a continuum of values, like the
O(2) model is due to the possibility of a wide domain wall that changes smoothly between the boundary states. In
the Ising case the domain wall is abrupt and this costs more energy. For the O(2) case, the Hamiltonian is,
X
X
~i S
~j = J
H = J
S
|S|2 cos(ij )
(13)
<ij>

<ij>

~i is a unit vector with angle i to the z axis, and ij = j i . To find the domain wall energy we consider
where S
the direction perpendicular to the domain wall and notice that the lowest energy domain wall is formed by making a
wide domain wall with small differences in energy between spins. Since the domain wall corresponds to changing the

3
spin orientation from up to down, the angle must rotate by . If the width of the domain wall is L, the angle between
adjacent spins is /L. The energy cost of the domain wall is then,
O(2)

EDW = Ld1 J

L
X
1

(1 cos( )) = Ld1 JL ( )2 = JLd2


L
2 L
2
i=1

(14)

The lower critical dimension is the dimension below which the domain wall energy is finite, while above the lower
critical dimension the energy of a domain grows with the length of the domain wall. From the above we find that for
the Ising system dlc = 1 + , while for the O(2) model dlc = 2 + , where  is small and positive.

Quiz Problem 7. Write down the van der Waals equation of state. Draw the P, v phase diagram of the van der
Waals gas and indicate the critical point.
Solution.
kb T
a

v b v2

P =

(15)

Quiz Problem 8. Make plots of the van der Waals equation of state isotherms, for T > Tc , T < Tc and for
T = Tc . For the case T < Tc explain why the non-convex part of the curve cannot occur at equilibrium and the
Maxwell construction to obtain a physical P, v isotherm.
Quiz Problem 9. Write down the Landau free energy for the Ising and fluid-gas phase transitions. Explain the
correspondences between the quantities in the magnetic and classical gas problems.
Solution.
F = a(T Tc )y 2 + by 4 + cy

(16)

For the Ising model y = m, c = h, for the van der Waals gas, y = vg vl , c = P Pc .

Quiz Problem 10. Derive the Helmholtz free energy of the van der Waals gas and explain the physical meaning
of the parameters a and b. Using your free energy explain the Maxwell construction.
Solution
See lecture notes.

Quiz Problem 11. Write down the Gibbs free energy of the van der Waals gas. Explain the conditions under
which co-existence is expected to occur.
Solution
See lecture notes

Quiz Problem 12. Explain the meaning of the upper critical dimension and lower critical dimension in the theory
of critical phenomena.
Solution
Below the lower critical dimension, there is no traditional phase transition to an ordered phase at finite temperature.
It should be noted however that at the critical dimension special transitions may occur, for example in the case of the
O(2) model the Kosterlitz-Thouless transition occurs in two dimensions and this is a special phase transition that is
different that the phase transitions of the model above the lower critical dimensions.
Above the upper critical dimension, the critical phenomena is correctly described by mean field critical exponents.
Between the upper and lower critical dimensions, critical exponents and the nature of the transition (continuous or
discontinuous) may change.

Quiz Problem 13. State the universality hypothesis in the theory of critical phenomena and using it explain why
the liquid gas phase transition is in the same universality class as the Ising mdoel.
Solution
The universality hypothesis refers to continuous phase transitions where the correlation length diverges so that
fluctuations occur on all length scales. The universality hypothesis states that the critical exponents describing
continuous phase transitions depend on only three things:
(i) The spatial dimension
(ii) The order parameter symmetry
(iii) The range of the interactions
This predicts for example that systems described by very different physics, such as the liquid-gas transition and
Ising magnetic phase transition, should be described by the same critical exponents.

Quiz Problem 14. Explain the importance of the linked-cluster theorems in perturbation theory of many
particle systems.
Solution.
Linked cluster theorems state that averages of observables can be described by sums of linked diagrams, and that
non-extensive contributions from disconnected diagrams must cancel when the diagrams are summed to all orders.

Quiz Problem 15. Draw the high temperature series expansion diagrams to order t8 (where t = tanh(J)) for
the square lattice, nearest neighbor, spin half Ising ferromagnet partition function. What is the degeneracy of each
of these diagrams? Write down the expansion for the Helmholtz free energy and give a physical reason why only the
terms of order N are kept.
Solution.
See Lecture Notes

Quiz Problem 16. Draw the low temperature series expansion diagrams to order s8 (where s = Exp[2J) for
the square lattice, nearest neighbor, spin half Ising ferromagnet partition function. What is the degeneracy of each
of these diagrams? Write down the expansion for the Helmholtz free energy and give a physical reason why only the
terms of order N are kept.
Solution.
See Lecture Notes

Quiz Problem 17. Write down the mathematical form of the virial expansion for many particle systems and
explain why it is important. What physical properties can be extracted from the second virial coefficient?
Solution
The virial expansion is given by,
P
= + B2 2 + B3 3 + ....
kB T

(17)

where = N/V is small. Keeping only the leading order term on the right hand side of this equation leads to the
ideal gas law. The first correction to the ideal gas law is due to the prefactor B2 , which is called the second virial
coefficient. The second virial coefficient for a particle system described by central force pair potential is given by,
Z
B2 = dR R2 (eu(r) 1)
(18)
From this expression we can see that the second virial coefficient is related to the interatomic potential. If we measure
B2 as a function of temperature it is possible to extract a lot of information about u(r).

Quiz Problem 18. Explain the meaning of second quantization. Discuss the way that it can be used in position
space and in the basis of single particle wavefunctions. Write down the commutation relations for Bose and Fermi
second quantized creation and annihilation operators.
Solution.
Second quantization is a formulation of quantum mechanics and of quantum field theory that is expressed in
terms of creation and annihilation operators. In many body quantum physics creation and annihilation operators
create and destroy particles in many body basis sets constructed from single particle wave functions. In the case
of Fermions a many body basis function is a determinant, while for Bosons it is a permanent. The commutation
relations for Fermions and Bosons are similar, except that for fermions we have anticommutators and for Bosons we
have commutators. In many body quantum mechanics we have,
[ai , aj ] = ij ;

f or bosons;

and

{ai , aj } = ij ;

f or bosons;

and

{(x), (x0 )} = (x x0 );

f or f ermions;

(19)

while for quantum fields, we have,


[(x), (x0 )] = (x x0 );

f or f ermions;

(20)

These days, new theories are often formulated using creation and annihilation operators rather than the Heisenberg
or Schr
odinger formulations of quantum theory.

Quiz Problem 19. Write down the Hamiltonian for BCS theory, and the decoupling scheme used to reduce it to
a solvable form. Explain the physical reasoning for the decoupling scheme that is chosen.
Solution.
In the s-wave BCS theory a singlet state is assumed, so that,
X
X
Hpair N =
(~k ) a~ a~k +
V~k~l a~ a ~ a~l a~l ,
k k,

~
k

where N =

~
k

(21)

~
k~l

n~k is the number of electrons in the Fermi sea. We carry out an expansion in the fluctuations,
a~l a~l = bl + (a~l a~l bl );

a~ a ~ = bk + (a~ a ~ bk )
k k

k k

The mean field Hamiltonian keeps only the leading order term in the fluctuations so that,
X
X
HM F N =
(~k ) a~ a~k +
V~k~l (a~ a ~ b~l + b~k a~l a~l b~k b~l)
k k,

~
k

(22)

(23)

~
k~l

This is the Hamiltonian that leads to the BSC solution.

Quiz Problem 20. Consider the inverse Bogoliubov-Valatin transformation,


~k = u~k a~k v~k a ~

(24)

Show that if the operators a, a obey standard fermion anti-commutator relations, then the operators , also obey
these relations, provided,
|uk |2 + |vk |2 = 1

(25)

Solution The anticommutator,


{~k , ~l } = {u~k a~k v~k a ~

, u~l a~l v~la ~

}.

(26)

6
Expanding the anticommutator gives,
{~k , ~l } = (u~k a~k v~k a ~

)(u~l a~l v~la ~

) + (u~l a~l v~la ~

)(u~k a~k v~k a ~

(27)

} v~k u~l {ak


, a~ }
~

(28)

which reduces to,


{~k , ~l } = u~k u~l {a~k , a~l } + v~k v~l{a~

, a ~

} u~k v~l{a~k , a ~

The first two anticommutators are zero. The second two anticommutators are finite when the conditions (k, l)(, )
hold. However when this condition holds, the two commutators are equal and opposite, so they sum to zero. Taking
the adjoint of Eq. (15) shows that {~k , ~l } = 0. Now consider,
{~k , ~ } = (u~k a~k v~k a ~

)(u~la~ v~l a~l ) + (u~la~ v~l a~l )(u~k a~k v~k a ~


l

{~k , ~ } = u~k u~l{a~k , a~ } + v~k v~l {a~


l

, a~l } u~k v~l {a~k , a~l } v~k u~l{a~

, a~ } = 0
l

(29)
(30)

which reduces to
{~k , ~ } = kl |u~k |2 {a~k , a~ } + |v~k |2 {a~
l

, a~l } = kl (|u~k |2 + |v~k |2 ).

(31)

This anticommutator thus requires Eq. (12) in order for the operators to obey Fermion anti commutator relations.

Quiz Problem 21. Given that the energy of quasiparticle excitations from the BCS ground state have the
spectrum,
E = [( F )2 + ||2 ]1/2 ,

(32)

where is the superconducting gap and EF is the Fermi energy, show that the quasiparticle density of states if given
by,
D(E) =

N (F )E
(E 2 2 )1/2

(33)

Solution We use the relation,


N (F )d = D(E)dE;

and

dE =

 F
d
[( F )2 + 2 ]1/2

(34)

to find,
D(E) =

N (F )[( F )2 + 2 ]1/2


 F

(35)

and using,
( F )2 = E 2 ||2

(36)

N (F )E
||2 ]1/2

(37)

yields,
D(E) =

[E 2

Quiz Problem 22. Describe the physical meaning of the superconducting gap, and the way in which BCS theory
describes it.
Solution
The superconducting gap is the energy required to generate a quasiparticle excitation from the superconducting
ground state. In BCS theory, the quasiparticles behave like non-interacting fermions and the energy required to
generate a quasiparticle is at least 2(T ).

Quiz Problem 23. Given the general solutions to the BCS mean field theory
~k =

X
~l

V~k~l

~l
,
2E~l

E~k = ((~k )2 + |~k |2 )1/2

(38)

Describe the assumptions that are made in deducing that,


Z F +hc
d
N (F )V
=
1=
2
2 1/2
2
F
hc (( F ) + || )
Z
N (F )V
0

h
c /

dx
hc
= N (F )V Sinh1 (
)

(1 + x2 )1/2

(39)

and hence,
= 2
hc Exp[

1
]
N (F )V

(40)

Solution
We assume an isotropic gap, and that the attractive coupling between electrons is constant V , over the range
F hc <  < F + hc . The density of states is assumed constant with value N (F ).

Assigned problems
Assigned Problem 1. From Eq. (18) of the notes, derive the Landau form Eq. (44). Explain the approximations
that are made. Plot FL as a function if m for T > Tc and T < Tc for h = 0 and for h 6= 0. Explain the concept of
spontaneous symmetry breaking (SSB) using your graphs.
Solution.
Equation (18) is,
fR =

FM F
1
+ ln(2) = Jzm2 + ln(Cosh(Jzm + h))]
N
2

1
1
1
Jzm2 + ((Jzm + h))2 ((Jzm + h))4 + O(m6 )
2
2
12

(41)

where we used the expansion of log(cosh(x)) for small x. Now we use the fact that kB Tc = Jz and expanding in h to
linear order we find,
fL = a(T Tc )m2 + bm4 + hm

(42)

where constant prefactors in the term hm are absorbed into fL , a and b.

Assigned Problem 2. Consider the Ising ferromagnet in zero field, in the case where the spin can take three values
Si = 0, 1. a) Find equations for the mean field free energy and magnetization. b) Find the critical temperature and
the behavior near the critical point. Are the critical exponents (, , , ) the same as for the case S = 1? Is the
critical point at higher or lower temperature than the spin 1 case? c) Is the free energy for the the spin 0, 1 case
higher or lower than the free energy of the 1 case? Why? d) Carry out an expansion of the free energy to fourth
order in the magnetization. Does this free energy have the Landau form expected for an Ising ferromagnet?
Solution. The partition function and Helmholtz free energy are,
Z = [1 + 2Cosh(Jzm)]N ;

F = kB T N ln[1 + 2Cosh(Jzm)]

(43)

8
The mean field equation is,
m=

2
1
2Sinh(Jzm)
Jzm (Jzm)3 + ...
1 + 2Cosh(Jzm)
3
3

(44)

The critical point is at Jz = 3/2, so the critical temperature is at kB Tc = 2Jz/3 which is lower than that for spin
1/2 due to the additional entropy of the spin one system. The critical exponents and are clearly the same as for
the spin 1/2 case. The free energy is lower for the spin 1 case due to the higher entropy.

Assigned Problem 3. By using b = show that Eqs. (58) reduces to Eqs. (50).
Assigned Problem 4. Consider the Landau free energy,
F = a(T Tc )m2 + bm4 + cm6

(45)

where c(T ) > 0 as required for stability. Sketch the possible behaviors for a(T ), b(T ) positive and negative, and show
that the system undergoes a first order transition at some value Tc . Find the value of a(Tc ) and the discontinuity in
m at the transition.
Solution.
The mean field equation is,
F
= am + bm3 + cm5 = 0;
m

(46)

Note that in general we do a variation with respect to m, so when we add fluctuations later, we need to use the
Euler-Lagrange equation. Here the variation is the same as a partial derivative with respect to m. Solving the mean
field equation, we find five solutions.
m = 0,

m = m ;

m2 =

b (b2 4ac)1/2
2c

(47)

Though there are always five solutions, only the real solutions are physical. Analysis of the behavior of the model
reduces to identifying the real solutions, and finding which real solution has the lowest free energy. We can understand
the nature of the solutions by looking at the second derivative,
2F
= a + 3bm2 + 5cm4 ,
m2

(48)

which enables us to distinguish between maxima and minima. We also use the fact that F is symmetric in m and
that at large m, because c is positive, F is large and positive for large |m|. Finally, without loss of generality, we can
divide through by c, or equivalently set c = 1. Now consider the four cases for a, b.
(i) a > 0, b > 0. In this case b2 4ac < b, so m2 is always negative. Therefore the solutions m are always
imaginary. The only real solution is m = 0, which is a minimum having F (0) = 0.
(ii) a < 0, b > 0. In this regime, b2 + 4|a|c > b2 , so m2+ > 0, so that m+ is real. However b (b2 + 4|a|c)1/2
remains negative, so m remains imaginary. The real solutions are thus a maximum at m = 0 and two symmetric
minima at m2+ = 21 ([b2 + 4|a|]1/2 b).
There is a phase transition between states (i) and (ii) that occurs at a = 0 where two new solutions emerge and the
extremum at m = 0 changes from a maximum for a < 0 to a minimum for a > 0. The nature of the transition is
found by making a small |a| expansion of the solutions m+ , which leads to m |a|1/2 . This is the Ising/Van der
Waals univesality class we have studied using mean field theory, where we found |a| |T Tc |
(iii) a < 0, b < 0. In this regime, b2 + 4|a|c > b2 , so m+ is real but m remains imaginary. Therefore, as in case
(ii), there is a maximum at m = 0, and minima at m+ .
(iv) a > 0, b < 0. In this regime there are several things going on. First, the discriminant b2 4a is negative for
b2 < 2a, so in this regime there is only one real solution, a minimum at m = 0. For b2 > 2a, there are five real
solutions because |b| (b2 4a)1/2 > 0. Moreover, we know that the solution at m = 0 is a minimum, so we know
that m are maxima, while m+ are minima.
The final issue we have to resolve is the behavior of the minima m+ as a function of a, b, in particular we need to
know if F (m+ ) is greater than or less than F (0). If the lowest free energy state changes it corresponds to a phase

9
transition. We can solve this problem by evaluating F (m+ ), or we can find solutions where F (m0 ) = 0 and then solve
m0 = m+ . The later leads to,
m20 =

b2
4a
3 |b|
[ ( )1/2 ]
2 2
4
3

(49)

and setting m20 = m2+ , we find,


b2
4a
1
3 |b|
[ ( )1/2 ] = [|b| + (b2 4a)1/2 ]
2 2
4
3
2

(50)

which has solution


a
|b| = 4( )1/2 ;
3

with

a
m2 = m20 (|b| , a) = 2( )1/2
3

(51)

m is the magnetization on the phase boundary defined by |b| . There is then a first order phase transition from
magnetization m for |b| < |b| , to magnetization m = 0 for |b| > |b| . The behavior of the magnetization on the
phase boundary is m a1/4 |T Tc |1/4 , which is the mean field result for the order parameter near a tricritical
point where a line of second order phase transitions meets a line of first order phase transitions.

Assigned Problem 5. Consider a Landau theory with a cubic term,


F = a(T Tc )m2 + bm3 + cm4 .

(52)

Analyse the behavior of this model, particularly the nature of the different phases and phase transition(s) that occur.
Solutions
It is convenient to write this in the form,
F =

2 3 4
m m + m ,
2
3
4

(53)

We require that > 0 to ensure that the magnetization at the free energy minimum is bounded, and we can take
= 1 without loss of generality. We also note that the transformation , m m leaves F invariant, so we
only consider > 0. First consider the case where = 0. In that case we have the Landau theory that we studied
before and we know that there is a continuous mean field phase transition at = 0 from a spontaneously ordered
phase for < 0(T < Tc ) to a disordered m = 0 phase for > 0(T > Tc ). For > 0, the positive magnetization state
is further stabilized, so we look for a transition at > 0. To determine the behavior for > 0, we find the extreme
of F ,
F
= 0 = m m2 + m3 ,
m
where has solutions,
m = 0; m =

p
1
[ 2 4]
2

(56)

when the disriminant is negative there is only one real solution (m = 0), while if the discriminant is positive, there
are three real solutions. The critical condition is called the spinodal line and it is given by,

s = 2 > 0
(57)
for < s there is no ordered phase, while for > s there are three extrema, but we dont yet know if there is a
stable ordered phase. To find out if a positive magnetization state is stable, we compare the free energies, f (0) and
f (m+ ) and find the critical condition by equating them,
f (0) = 0 =

1
m+ m3+ + m4+
2
3
4

(58)

10
or,
1

m+ + m2+ = 0
2
3
4

(59)

m2+ = m+

(60)


1
m+ + (m+ ) = 0
2
3
4

(61)

We also have,

so that,

or
m+ +

3
= 0.

(62)

Using the solution for m+ , we find,


p

2 4 =

(63)

Solving yields,
3
c = .
2

(64)

For < c , the model predicts an ordered phase with finite magnetization, and at c there is a first order transition
into a phase with finite magnetization. In the regime s < < c the system has zero magnetization but there is a
metastable state with finite magnetization. For < s the zero magnetization state is the only local minimum in the
model.

Assigned Problem 6.
A spin half Ising model with four spin interactions on a square lattice has Hamiltonian,
X
H=
JSi Sj Sk Sl

(65)

ijkl in square

where the sum is over the smallest squares on an infinite square lattice, the interaction is ferromagnetic J > 0 and
Si = 1. Each elementary square is counted only once in the sum.
Using a leading order expansion in the fluctuations (i.e. write Si = mi + (Si mi ) and expand to leading order in
the fluctuations), find the mean field Hamiltonian for this problem (Here mi =< Si > is the magnetization at site i).
Using the mean field Hamiltonian and assuming a homogeneous state where mi = m, find an expression for the
mean field Helmholtz free energy, and the mean field equation for this problem.
Taking J = 1, sketch the behavior of the solutions to the mean field equation as a function of temperature. Does
the non-trivial solution move continuously toward the m = 0 solution as the temperature increases? Is the behavior
of the order parameter at the critical temperature discontinuous or continuous? Do you expect the correlation length
to diverge at the critical point in this problem?
Solutions
We start by writing a three spin term in terms of an expansion in the fluctuations,
Si Sj Sk Sl = [mi + (Si mi )][mj + (Sj mj )][mk + (Sk mk )][ml + (Sl ml )]

(66)

Expanding to leading order in the fluctuations, we then have,


Si Sj Sk Sl mi mj mk ml + (Si mi )mj mk ml + mi (Sj mj )mk ml + mi mj (Sk mk )ml + mi mj mk (Sl ml ) (67)

11
Since the interaction is ferromagnetic we can make the uniform assumption mi = m, which yields,
X
X
HM F =
[3Jm4 + m3 (Si + Sj + Sk + Sl )] = 3JN m4 4Jm3
Si

(68)

ijkl in square

The mean field partition function is then,


4

ZM F = e3JN m [2cosh(4Jm3 )]N

(69)

FM F = kB T lnZM F = 3JN m4 kB T N ln[2cosh(4Jm3 )]

(70)

which leads to the free energy,

and the mean field equation is,


m = tanh(4Jm3 )

(71)

The mean field equation can be found in several ways, for example by doing the variation F/m = 0, or by finding
the expectation < Si > using the mean field Hamiltonian.
By plotting the graphs of m and tanh(4Jm3 ), it is evident that at large values of K = J (i.e. low temperature),
there are two positive m solutions as well as the solution at m = 0. However as K decreases (i.e. T increases) there is
a critical point at which the two non-trivial positive m solutions merge. This defines the critical point. At this point
m is finite. For T > Tc the positive m solution disappears so there is a discontinuous jump in the magnetization from
a finite value to zero at Tc . The transition in this model is then first order, in contrast to the pair interaction case
where the transition is continuous. The correlation length at the transition found here is then remains finite at the
critial point.
Assigned Problem 7. The Dieterici equation of state for a gas is,
P =

kB T a/(kB T v)
e
vb

(72)

where v = V /N . Find the critical point and the values of the exponents , , for this model.
Solution. The critical point is found by solving,
Pc =

kB Tc a/(kB T vc )
e
;
vc b

P
kB Tc a/(kB T vc )
kB Tc
a
=0=
e
+
ea/(kB T vc )
v
(vc b)2
vc b kB T vc2

(73)

(74)

Which simplify to,

1
a
+
= 0;
vc b kB Tc vc2

so

a
vc2
=
kB Tc
vc b

(75)

The second derivative yields,


2P
kB Tc a/(kB T vc )
kB T
a
=0=2
e
2
ea/(kB Tc vc )
2
3
2
v
(vc b)
(vc b) kB Tc vc2

a
kB Tc
a
kB Tc
ea/(kB Tc vc ) +
(
)2 ea/(kB Tc vc )
3
vc b kB Tc vc
vc b kB Tc vc2

(76)

1
1
a
a
a
2
2
+(
)2 = 0
2
2
3
(vc b)
vc b kB Tc vc
kB Tc vc
kB Tc vc2

(77)

so that,
2

12
and using Eq. (),
2

1
1 1
+(
)2 = 0;
vc vc b
vc b

so

vc = 2b

(78)

so we find that,
vc = 2b;

kB Tc =

a
;
4b

Pc =

a
4e2 b2

(79)

To find the critical exponents we write v = vc + v, T = Tc + T, P = Pc + P , so that,


Pc + P =

kB (Tc + T ) a/[kB (Tc +T )(vc +v)]


kB Tc (1 +
e
=
vc + v b
b
1+

T
Tc
v
b

ea/[kB Tc vc (1+T /Tc )(1+v/vc )]

(80)

This reduces to,


1 + p = e2

1+t
2
Exp[
]
1 + 2x
(1 + t)(1 + x)

(81)

where p = P/Pc , t = T /Tc , x = v/vc . To third order this expansion gives,


2
2
p = 3t + 2t2 t3 + (2t 4t2 )x + 2tx2 x3
3
3

(82)

Taking a derivative with respect to v at setting x = 0 leads to,


T = (V (

P
)T )1 |T Tc |1
V

(83)

so = 1. The exponent is found by setting t to zero so that p x3 , so that = 3. To find we assume that
pl = pg , xl = xg , so that,
2
2
pl = 3t + 2t2 t3 + (2t 4t2 )xl + 2tx2l x3l
3
3

(84)

2
2
p g = 3t + 2t2 t3 (2t 4t2 )xl + 2tx2l + x3l
3
3

(85)

Setting these equations to be equal yields,


2(2t 4t2 )xl =

2 3
x ,
3 l

so that

xl |T Tc |1/2

(86)

where we dropped the t2 term as it is higher order. In the above analysis the signs of the t and x are consistent but
have to be checked each time.

Assigned Problem 8. Consider a phase co-existence curve in a P T phase diagram, separating two phases
A and B. Consider two points on the phase coexistence curve at P, T and P + P, T + T . Since the chemical
potential of the phases A and B are the same at any given point on the co-existence curve, we have,
gA = gA (P + P, T + T ) gA (P, T ) = gB (P + P, T + T ) gB (P, T ) = gB

(87)

From this relation, prove the Clausius-Clapeyron relation,


P
L
=
T
T (VB VA )

(88)

where L is the latent heat. Find the form of this relation for the van der Waals equation of state. What is the
dependence of the latent heat as T Tc . Is this exponent related to any of the other exponents in the problem?

13
Solution
The Gibbs energies G(B), G(A) are the same at temperature T and at temperature T + T , so dGA = dGB , where
dG = G(T + dT ) G(T ). We also have,
dG = SdT + V dP ;

dG
dP
= S + V
dT
dT

so

(89)

Using the latter expression for both phases we have,


SA + VA

dP
dP
= SB + VB
dT
dT

(90)

where we used the fact that the pressure is the same in the A and B phases. Now we use the Clausius relation
dS = dQ/T , and the fact that dQ = L to find,
dP
SB SA
L
=
=
dT
VB VA
T (VB VA )

(91)

Assigned Problem 9. Recent work on black hole thermodynamics has suggested that a black hole with charge
Q obeys the equation of state,
P =

T
1
2Q2

+
2
v
2v
v 4

(92)

where the physical pressure and temperature are given by,


P ress =

c
h
;
lP2

T emp =

hc
T;
kB

v = 2lP2 r+

(93)

where lP = hGN /c3 is the Planck length and r+ is the black hole event horizon. Yes, the Temp temperature
expression looks strange, but that is the expression in the paper I got this from and I have not had time to figure
out if there is a typo in the paper. In any case you dont need to use that equation. Find the critical point of this
equation of state and find the critical exponents and .
Solution
The conditions for the critical point are,
2P
P
=
=0
v
v 2

(94)

which lead to the two equations,

T
1
8Q2
+ 3
= 0;
2
v
v
v 5

2T
3
40Q2
4+
=0
3
v
v
v 6

(95)

3v 2
40Q2
+
=0

(96)

which reduce to,


T v 3 +

v2
8Q2

= 0;

2T v 3

Multiplying the first equation by two and adding it to the second equation gives,

v2
24Q2
+
= 0;

so

vc =

Pc =

1
96Q2

24Q

(97)

and hence,
1
Tc =
;
3 6Q

(98)

Defining p = P/Pc , = v/vc , t = T /Tc , we find,


8t = 3(p +

2
1
) 3
2

(99)

14
In these variables the critical point p, t, v is at 1, 1, 1. To find the critical exponents we expand as in the van der Waals
case, so we define, t = 1 t, = 1 + ; p = 1 + p.
1
2
8 (1 t)

= 1 + p +
2
3 1 +
(1 + )
3(1 + )4

(100)

1
1
1
= 1 lx + l(l + 1)x2 l(l + 1)(l + 2) + ...
(1 + x)l
2
3!

(101)

Using the Taylor expansion,

we find,
8
1
(1 t)(1 v + v 2 v 3 + ...) = 1 + p + 2(1 2v + 3v 2 4v 3 + ...) (1 4v + 10v 2 20v 3 + ...) (102)
3
3
Expanding gives,
8
8
8
4
8
(1 v + v 2 v 3 t + tv tv 2 + ..) = + p v + v 2 v 3
3
3
3
3
3

(103)

4
p = t + tv tv 2 v 3
3

(104)

which reduces to,

Using the fact that vg = vl and subtracting equations for both the gas and liquid phases leads to,

vg = 0.75t1/2 .

(105)

Finally setting t = 0, we find vg p1/3 . We thus find the exponents = 1/2 and = 3 as expected for mean field
systems.

Assigned Problem 10. Consider a ferromagnetic nearest neighbor, spin 1/2, square lattice Ising model where the
interactions along the x-axis, Jx , are different than those along the y-axis, Jy . Extend the low and high temperature
expansions Eq. (150) and Eq. (152) to this case. Does duality still hold? From your expansions, find the internal
energy and the specific heat.
Solution
Z=

{Si =1} <ij>

eKij Si Sj = (Cosh(Kx ))N (Cosh(Ky ))N

(1 + tij Si Sj ).

(106)

{Si =1} <ij>

tij = tx for horizontal bonds and tij = ty for vertical bonds, where tx = tanh(Kx ), ty = tanh(Ky ). The diagrams
used are similar, but now we have to treat subclasses with different numbers of hirizontal and vertical bonds, so that,
F = ln(Z) =

9
zN
ln(Cosh(K)) + N ln(2) + ln[1 + N t4 + 2N t6 + N (N + )t8 + 0(t10 )]
2
2

(107)

becomes
5
F = N [ln(2) + ln(Cosh(Kx )) + ln(Cosh(Kx )) + t2x t2y + t2x t2y (t2x + t2y ) + (t4x t4y ) + t2x t6y + t6x t2y + ....]
2
Similarly, the extension of the low temperature expansion,
X Y
9
Z=
eKSi Sj = eKzN [1 + N s4 + 2N s6 + N (N + )s8 + O(s10 )]
2
<ij>

(108)

(109)

{Si =1}

to the anisotropic case leads to,


5
F = N [Kx + Ky + s2x s2y + s2x s2y (s2x + s2y ) + s4x s4y + s2x s6y + s6x s2y ...]
2
where sx = e2Kx , sy = e2Ky . Duality holds for both x and y directions.

(110)

15

Assigned Problem 11. Find the second virial coefficient for four cases: (i) the classical hard sphere gas; (ii)
Non-interacting Fermions; (iii) Non-interacting Bosons; (iv) The van der Waals gas.
Solution

X
Pv
3
=
al (T )( )l1
kB T
v

(111)

l=1

where the first virial coefficient a1 (T ) = 1, and the second virial coefficient is a2 (T ). The virial expansion is most
often carried out in the grand canonical ensemble, where we may write,

P
1 X l
= 3
bl z ;
kB T

l=1

N
1 X
= 3
lbl z l
V

(112)

l=1

so that,
P

l
X
3 l1
Pv
l=1 bl z
=
= P
a
(T
)(
)
l
l
kB T
v
l=1 lbl z

(113)

l=1

which gives relations between the quantities al (T ) and bl (T ). For the second virial coefficient the relationship is
a2 (T ) = b2 (T ). We have already calculated the coefficients bl for the ideal Bose and Fermi gases, with the results,
bl =

(1)l+1
;
l5/2

(Ideal F ermi)

bl =

l5/2

(Ideal Bose)

(114)

so we have,
Pv
1 3
= 1 5/2 ( ) + ....;
kB T
v
2

(Ideal Bose)

(115)

(Ideal F ermi)

(116)

and
Pv
1 3
= 1 + 5/2 ( ) + ....;
kB T
v
2

The van der Waals equation of state may be expanded to find the second virial coefficient,
P =

a2
kB T
,
vb v

so that

Pv
1
a
b a/kB T
=

1+
+ ...
kB T
(1 b/v) kB T v
v

For the classical interacting gas, b2 is given by,


Z
Z
1
1
r 3
u(~
r1 r~2
b2 =
d1 d r2 [e
1] = 3
d3 r[eu(~r) 1]
2V 3
2

(117)

(118)

For the hard sphere problem, with a hard core radius of R, we then find that b2 = 2R3 /(33 ), so the virial
expansion for this case is,
Pv
2R3
=1+
+ ....
kB T
3v

(119)

The behavior of the second virial coefficient as a function of temperature can be used to deduce the interaction
potential, and the importance of quantum effects as they have different temperature dependences.
Assigned Problem 12. The BCS pairing Hamiltonian is a simplified model in which only pairs with zero center of
mass momentum are included in the analysis. We also assume that the fermion pairing that leads to superconductivity
occurs in the singlet channel. The BCS Hamiltonian is then,
X
X
Hpair N =
(~k ) a~ a~k +
V~k~l a~ a ~ a~l a~l ,
(120)
k k,

~
k

~
k~l

16
where N =
defining,

~
k

n~k is the number of electrons in the Fermi sea. By making an expansion in the fluctuations and
b~k =< a~k a~k >, and b~k =< a~ a ~ > .

(121)

k k

where b~k is the average number of pairs in the system at wavevector ~k, show that the mean field BCS Hamiltonian is
given by,
X
X
HM F N =
(~k ) a~ a~k +
V~k~l (a~ a ~ b~l + b~k a~l a~l b~k b~l)
(122)
k k,

~
k

~
k~l

This is the Hamiltonian that we will solve to find the thermodynamic behavior of superconductors, using an atomistic
model.

Assigned Problem 13. The BCS pairing Hamiltonian is a simplified model in which only pairs with zero center of
mass momentum are included in the analysis. We also assume that the fermion pairing that leads to superconductivity
occurs in the singlet channel. The BCS Hamiltonian is then,
X
X
Hpair N =
(~k ) a~ a~k +
V~k~l a~ a ~ a~l a~l ,
(123)
k k,

~
k

where N =

~
k

~
k~l

n~k is the number of electrons in the Fermi sea. Defining,


b~k =< a~k a~k >, and b~k =< a~ a ~ > .

(124)

k k

carry out a leading order expansion in fluctuations, leading to,


X
X
HM F N =
(~k ) a~ a~k +
V~k~l (a~ a ~ b~l + b~k a~l a~l b~k b~l)
k k,

~
k

(125)

~
k~l

This is the Hamiltonian that we will solve to find the thermodynamic behavior of superconductors, using an atomistic
model.
Solution. The mean-field Hamiltonian can be considered as a first order expansion in the fluctuations i.e.
a~k a~k =< a~k a~k > +[a~k a~k < a~k a~k >]

(126)

Substition of this into the Hamilonian, along with the definitions above lead to HM F .

Assigned Problem 14. Using the Bogoliubov-Valatin transformation (Eq. 163), show that the mean field BCS
Hamiltonian (Eq. (162)) reduces to Eq. (166), provided Equations (167) and (168) are true.
Solution. With this transformation, the mean field Hamiltonian looks messy,
HM F N =
X

(~k ) (a~ a~k + a~ a~k )


k

~
k

(~k a~ a ~ + ~k a~k a~k b~k ~k ) =

k k,

~
k

~
k

(~k ) ([u~k ~ + v~k ~k ][u~k ~k + v~k ~ ] + [u~k ~ v~k ~k ][u~k ~k v~k ~ ])


k

~k [u~k ~ + v~k ~k ][u~k ~ v~k ~k ] ~k [u~k ~k v~k ~ ][u~k ~k + v~k ~ ] + b~k ~k


k

17
Expanding this yields,
X
(~k )[u~k u~k ~ ~k + u~k v~k ~ ~ + v~k u~k ~k ~k + v~k v~k ~k ~ ]
k k

~
k

+(~k )[u~k u~k ~ ~k u~k v~k ~ ~ v~k u~k ~k ~k + v~k v~k ~k ~ ]


k k

~k [u~k u~k ~ ~ u~k v~k ~ ~k + v~k u~k ~k ~ v~k v~k ~k ~k ]


k k

~k [u~k u~k ~k ~k + u~k v~k ~k ~ v~k u~k ~ ~k v~k v~k ~ ~ ] + b~k ~k


k

k k

We now collect the terms in this expression into three catagories: Those which have no operators in them; those which
can be reduced to diagonal form ie. those which are of the form k k and; those that are off diagonal (e.g. ).
The first stage is to collect together the terms which look to be in these three catagories. First the constant term,
X
HM F N =
b~k ~k
~
k

The following terms can be converted to diagonal form,


+(~k ) (|u~k |2 ~ ~k + |v~k |2 ~k ~ + |u~k |2 ~ ~k + |v~k |2 ~k ~ )
k

~k [u~k v~k ~ ~k + v~k u~k ~k ~ ] ~k [u~k v~k ~k ~ v~k u~k ~ ~k ]


k

Finally the off-diagonal terms are,


+(~k ) (u~k v~k ~ ~ u~k v~k ~ ~ + v~k u~k ~k ~k v~k u~k ~k ~k )
k k

k k

~k [(u~k )2 ~ ~ v~k2 ~k ~k ] ~k [u~2k ~k ~k (v~k )2 ~ ~ ]


k k

k k

We have to rearrange the terms catagorized as diagonal above, as we need them in the form . We do this using
the commutation relation, i.e. = 1 . This yields,
X
HM F N =
b~k ~k + 2(~k )|v~k |2 ~k u~k v~k ~k u~k v~k
~
k

+(~k ) (|u~k |2 ~ ~k |v~k |2 ~ ~k + |u~k |2 ~ ~k |v~k |2 ~ ~k )


k

+~k [u~k v~k ~ ~k + v~k u~k ~ ~k ] + ~k [u~k v~k ~ ~k + v~k u~k ~ ~k ]


k

Finally the off-diagonal terms are (as before),


+(~k ) (u~k v~k ~ ~ u~k v~k ~ ~ + v~k u~k ~k ~k v~k u~k ~k ~k )
k k

k k

~k [(u~k )2 ~ ~ v~k2 ~k ~k ] ~k [u~2k ~k ~k (v~k )2 ~ ~ ]


k k

k k

We note that n~k = n~k , and that the expectation of other terms that transform into one another through the
transformation ~k ~k, are equivalent. Collecting terms then leads to Eqs. (166)-(168) of the lecture notes.

18
Assigned Problem 15. We define
v~k =

g~k
(1 + |g~k |2 )1/2

(127)

show that Eq. (167) reduces to (173).


Solution Starting with,
2(~k )(1 |v~k |2 )1/2 v~k + ~k v~k2 ~k (1 |v~k |2 ) = 0,

(128)

1
.
(1 + |g~k |2 )1/2

(129)

We have,
|u~k |2 = 1 |v~k |2 =

1
;
1 + |g~k |2

uk =

Substitution into Eq. () leads to,


2(~k )g~k + ~k g~k2 ~k = 0.

(130)

Assigned Problem 16. Show that E~k as defined in Eq. (174) is in agreement with Eq. (168).
Solution We need to show that the definitions,
E~k = (~k )(|u~k |2 |v~k |2 ) + ~k u~k v~k + ~k v~k u~k .

(131)

E~k = [(~k )2 + |~k |2 ]1/2

(132)

and

are the same. To do this we use the results of Problem 9 in Eq. (53) to write,
E~k = (~k )(

~k
[E~k (~k )]2
[E~k + (~k )]2
~

)
+

+ ~k k .
~
2
2
k
4E~
4E~
2E~k
2E~k
k

(133)

which reduces to Eq. (54) as required.

Assigned Problem 17. Prove the relations Eq. (175-177).


Solution We have,
g~k =

E~k (~k )
;
~k

E~k = [(~k )2 + |~k |2 ]1/2

(134)

so that,
|~k |2 = E~k2 (~k )2 ;

(135)

so that,
|g~k |2 =

[E~ (~k )]2


E~ (~k )
E~k (~k )
= k2
= k
2
~k
E~ (~k )
E~k + (~k )

(136)

|g~k |2
E~ (~k )
;
= k
2
1 + |g~k |
2E~k

(137)

We then find,
|v~k |2 =

|u~k |2 =

E~k + (~k )
2E~k

and hence,
u~k v~k =

~k
g~k
2
=
g
|u
|
=
~
~
k
k
1 + |g~k |2
2E~k

(138)

Solutions to assigned problems and sample quiz problems for Part 4


Sample Quiz Problems
Quiz Problem 1. Write down the BCS gap equation at finite temperature and compare it to the zero temperature
expression. Explain the origin of the differences between the two expressions.
Quiz Problem 2. Make a plot of the BCS gap as a function of temperature. Write down the scaling behavior of
the gap on approach to the critical temperature.
Quiz Problem 3. Starting from the quantum mechanical expression for the current, show that in neutral superfluids circulation is quantized. Give an expression of the quantum of circulation.
Quiz Problem 4. Starting from the quantum mechanical expression for the current in the presence of a vector
potential, show that flux quantized. Give an expression for the flux quantum.
Quiz Problem 5. Describe the physical meaning of the healing length ( ) in superfluids and superconductors.
By considering the linearized Ginzburg-Landau equation in zero field find a solution describing the attenuation of
superconducting pair density near the surface of a superconductor.
Quiz Problem 6. Using either Londons original argument or starting with the Ginzburg-Landau equation, derive
the London differential equation describing the penetration of parallel magnetic field into a superconducting surface.
Show that it has the solution B(x) = B0 ex/ , where is the London penetration depth.
Quiz Problem 7. What is the mixed phase of a type II superconductor? Give a physical reasoning to explain
why the mixed phase of a type II superconductor can have, at sufficienty high external field, a lower free energy than
the Meissner state.
Quiz Problem 8. Using Londons theory, find an expression for the lower critical field of a type II superconductor.
Quiz Problem 9. Using G-L theory find an expression for the upper critical field of a type II superconductor.
Quiz Problem 10. Write down the Langevin equation for motion of a particle in the presence of random forces.
Describe the physical meaning of the random forces and the meaning of Gaussian white noise.
Quiz problem 11. Write down the non-conserved order parameter relaxation equation for the Ising model within
Landau theory and show that near the critical point the relaxation time diverges.
Quiz problem 12. Explain the difference between conserved dynamics and non-conserved dynamics. Give an
example of the two cases.
Quiz problem 13. Sketch the phase diagrams of the van der Waals gas and the ferromagnetic Ising model,
indicating the spinodal lines. How are the spinodal lines defined in the two cases.
Quiz problem 14. Explain the physical basis for the concept of a critical droplet in the homogeneous nucleation
of a new stable phase from a metastable phase. Find an expression for the critical droplet size for an Ising model in
a magnetic field at low temperature.
Quiz problem 15. Derive the Vlasov equation for the time evolution of the single particle distribution function
f (t, ~r, ~v ) for a collisionless system.

Assigned problems
Assigned Problem 1. By doing a variation with respect to of the Ginzburg-Landau free energy (Eq. (64)) of
the notes, derive the Ginzburg-Landau equation (65).
Solution
Within Ginzburg-Landau theory, the Gibbs free energy for a superconductor is given by,

d3 r

gGL =


b(T )
B2
0 H 2
1
|(i
h + qA)(~r))|2 + a(T )|(~r)|2 +
|(~r)|4 +
+
BH .
2m
2
20
2

(1)

Expanding this expression(and dropping function arguments), we have,




Z
1
b 2 2 ( A)2
0 H 2
3

gGL = d r
[(i
h + qA) (i
h + qA)] + a + ( ) +
+
( A) H . (2)
2m
2
20
2
Expanding the first term yields,
Z
gGL =

3

1 X 2

[
h
+ ihqA (

) + q 2 A2 ]
2m =1 x x
x
x

d3 r

( A)2
0 H 2
b
+
( A) H
+ a + ( )2 2 +
2
20
2


(3)

Using ~r = (x1 , x2 , x3 ), the Euler-Lagrange equation for a variation with respect to is,
gGL X
gGL
gGL
=

( )


x
(

x )

(4)

gGL
1 X

1 X

=
(ihqA
+ q 2 A2 ) + a + b||2
(h2
ihqA ).

2m
x
2m x
x

(5)

Carring out this variation yields,

This is equivalent to,


1
gGL
=
(ih + qA)2 + a + b||2

2m

(6)

which is the GL equation in an applied field, as required. Another useful form is found by using the London gauge
where A = 0 to find,
1 X
gGL
2

=
(
h2 2 + 2ihqA
+ q 2 A2 ) + a + b||2 = 0

2m
x
x

(7)

Assigned Problem 2. By doing a variation with respect to the vector potential A of the Ginzburg-Landau free
energy (Eq. (64)) of the notes, derive the expression for the current (66).
Solution
The Euler-Lagrange equation for a variation with respect to one component Ai of the vector potential is,
gGL
gGL X
gGL
=

( Ai )
Ai
Ai
x
( x )

(8)

We then have,
X 1 ( A)2
gGL
hq

q2
=i
(

) + Ai
(
)
Ai
Ai
2m
xi
xi
m
20 x ( x
)

(9)

The term B H does not contribute as it is linear in the derivatives of A and hence is zero after the application of
the second term in the Euler-Lagrange equation. The expansion of ( A)2 is,
( A) ( A) = (

A3
A2 2
A1
A3 2
A2
A1 2

) +(

) +(

)
x2
x3
x3
x1
x1
x2

(10)

3
Using this expression to evaluate the last term in the case Ai = A1 , yields
X ( A)2
2 A2
2 A1
2 A3
2 A1
)
=
2(
+

(
+
)
1
x ( A
x1 x2
x22
x23
x1 x3
x )

(11)

Similar expressions hold for A2 and A3 . Now note that ( A) = ( A) 2 A. The first element of this
expression is,
2 A2
2 A1
2 A1
2 A1
2 A3
2 A1
+

x21
x1 x2
x1 x3
x21
x22
x23

(( A) 2 A)1 =
We thus find the identity,

( A)2
= 2 ( A) = 2[( A) 2 A]
A

(12)

( A) = B = 0 J

(13)

We also have,

Substituting these results into the equation above yields the simple result,

X 1 ( A)2
) = js
(
Ai
20 x ( x
)

Using this in equation (9) for we find,


js = i

q2
q
h
( ) Ai
2m
m

(14)

as required.

Assigned Problem 3. Show that minimizing the Helmholtz free energy within London theory,
Z
1
f1 =
(B 2 + 2 20 J 2 )2rdr
20 0

(15)

gives the London equation.


Solution
We apply the Euler-Lagrange equations to,
fLondon

1
=
20

dV [2 ( B)2 + B 2 ].

(16)

Using the Euler-Lagrange equation with the identities


( B)2
= 2 ( B) = 2[( B) 2 B]
B

(17)

and using B = 0, yields the London equation 2 2 B = B.

Assigned Problem 4. Using London theory, find the Helmholtz free energy per unit length of a vortex in a
superconductor in an external field H.
Solution. The energy(per unit length) of an isolated vortex is given by,
Z
B2
1
1 =
(
+ s vs2 )2rdr
2
2
0
0

(18)

4
The first term is the field energy and the second is the kinetic energy of the superconducting electrons (see Eq. (4)).
The supercurrent is related to the velocity by,
J = nb qvs

(19)

and using Maxwells equation, we then write,


1 =

1
20

(B 2 + 2 20 J 2 )2rdr

(20)

which explicitly shows the contributions of the field and the current. In the large / limit, this is dominated by the
regime r , so we find an approximate value of the vortex energy by using,
Z
1
0 2

1
)
(
2
[r(Ln( ))2 + r( )2 ]dr
20 22
r
r

20
=
40 2

[x(Ln(x))2 +

1
]dx
x

x2 1
20
[ ( Ln(x) + Ln(x)2 ) + Ln(x)]|1
2

40 2 2

20

[Ln( )]
2
40

(21)

Notice that the energy cost of forming the vortex is dominated by the kinetic energy of the superconducting electrons
(the logarithmic term). The energy cost due to the magnetic field is relatively small.

Assigned Problem 5. Using the Gibbs free energy within London theory, demonstrate that the triangular lattice
vortex array has lower energy than the square lattice array, for a fixed applied field, H > Hc1 , which is close to Hc1 .
Solution
The energy to add the N th vortex to a vortex lattice is given by the energy of the added vortex plus the vortexvortex interactions. Within London theory, the Gibbs free energy (or chemical potential) to add the N th vortex to a
vortex array is given by,
X B0 0
g = (1 0 H) +
K0 (|~r ~rj |/)
(22)
0
j
Using Hc1 = 1 /0 , the relation between the applied field and the lattice spacing is found by setting g = 0, which
yields,
B0 X
K0 (|~ri ~rj |/)
(23)
H Hc1 =
0 N ij
This fixes the lattice spacing, a, of a flux array. This spacing is different for different lattice structures. Associated
with the lattice spacing there is a total number of vortices inside the superconductor. This total number of vortices,
at fixed a, is larger for the triangular lattice, Ntr than it is for the square lattice, Ntr > Nsq . In fact the highest
packing possible is for the triangular lattice case.
In setting up a vortex lattice, the gain in Gibbs free energy is given by,
X B 0 0
G = N (1 0 H) +
K0 (|~ri ~rj |/)
(24)
20
ij
Notice that there is a factor of two in the second term(compared to Eq. (33)), which is the energy cost of vortex-vortex
repulsion, due to the fact that the energy cost in setting up the flux lattice is small for the first vortices added to the
lattice. Using Eq. (34), this can be rewritten as,
0

G
1
= (Hc1 H) + (H Hc1 );
N
2

so that

G
1
=
(Hc1 H)
N
20

(25)

5
This is the energy gain per vortex in setting up the lowest energy vortex lattice. Notice that if we consider only
nearest neighbor interactions, the energy per vortex is the same no matter what the vortex lattice structure. However
the number of vortices which can be added to the superconductor does depend on the structure of the final vortex
lattice. Since the number of vortices in the case of a square lattice is smaller than the number added for the triangular
lattice, the total energy is lowest for the case of a triangular lattice. Notice that this result is general, the triangular
lattice is the true ground state because it has the highest possible vortex density.

Assigned Problem 6. Go through the calculations to show that the kinetic energy and angular momentum per
unit length of a quantum of circulation in a neutral superfluid are given by equations (40) and (41) of the notes.
Solution We have,
b

Z
kin =

1
dm(r)vs (r)2 =
2

20 s
1
(0 )2
=
s (2rdr)
ln(b/)
2
(2r)2
4

(26)

and
Z
lkin =

vs (r)
=
r dm(r)
r
2

s (2r3 dr)

0
0 s 2
=
(b 2 )
2r
2

(27)

Assigned Problem 7. Considering only the kinetic energy of the supercurrent and using arguments like those
of Problem 6, show that the supercurrents circulating around a flux quantum in a superconductor have energy
approximately given by Eq. (55) of the notes.
Solution Using the same procedure as for the neutral superfluid, with vs = mjs /(qs ) with current given by,

js (r) = j0 ;
r

j0 =

0
20 3

(28)

we find,
Z
kin =

1
dm(r)vs (r)2 =
2

2
1
m0
20
2
s (2rdr)(
)
=
ln(/)
2
2qs 0 3 r2
40 2

(29)

where we used,
=(

m2 1/2
)
s q 2 0

(30)

Assigned Problem 8. Derive the drift diffusion equation


~ + eDn n ;
J~n = enn E
x

~ eDp p
J~p = epp E
x

from the Boltzmann equation using the relaxation time approximation.


Solution
Same as the solution for J~n with a change in sign in the force and in the definition of the current.

(31)

6
Assigned Problem 9. The collision integral (Eq. (113) of the notes) is zero when,
f (~
p1 )f (~
p2 ) = f (~
p3 )f (~
p4 ).

(32)

Show that the Maxwell Boltzmann distribution


~

f (t, ~r, ~v ) = Ce(~vV )

(33)

satisfies this equation, for the case of elastic scattering.


Solution
Subsituting into the equation, cancelling C factors and taking a logarithm of both sizes, then cancelling factors
gives,
~ )2 + (~v1 V
~ )2 = (~v3 V
~ )2 + (~v4 V
~ )2
(~v1 V

(34)

~
|~v1 |2 + |~v2 |2 (|~v3 |2 + |~v4 |2 ) = 2[(~v1 + ~v2 ) (~v3 + ~v4 )] V

(35)

Expanding and rearranging gives,

The left hand side is zero for elastic scattering, while the right hand side is zero by momentum conservation, hence
the Maxwell-Boltzmann form leads to zero for the collision integral in this case.

You might also like